JUMC Pharm III

Ace your homework & exams now with Quizwiz!

Questions 1 and 2. A 13-year-old boy with type 1 diabetes is brought to the hospital complaining of dizziness. Laboratory findings include severe hyperglycemia, ketoacidosis, and a blood pH of 7.15. 1. Which of the following agents should be administered to achieve rapid control of the severe ketoacidosis in this diabetic boy? (A) Crystalline zinc insulin (B) Glyburide (C) Insulin glargine (D) NPH insulin (E) Tolbutamide 2. Which of the following is the most likely complication of insulin therapy in this patient? (A) Dilutional hyponatremia (B) Hypoglycemia (C) Increased bleeding tendency (D) Pancreatitis (E) Severe hypertension

1. Oral antidiabetic agents (listed in Table 41-1) are inappropriate in this patient because he has insulin-dependent diabetes. He needs a rapid-acting insulin preparation that can be given intravenously (see Figure 41-1). The answer is A. 2. Because of the risk of brain damage, the most important complication of insulin therapy is hypoglycemia. The other choices are not common effects of insulin. The answer is B.

A 65-year-old woman has returned from a vacation abroad suffering from traveler's diarrhea, and her problem has not responded to antidiarrheal drugs. A pathogenic gram-negative bacillus is suspected. Which drug is most likely to be effective in the treatment of this patient? (A) Ampicillin (B) Ofloxacin (C) Sulfadiazine (D) Trimethoprim (E) Vancomycin

2. The second-generation fluoroquinolones are very effective in diarrhea caused by bacterial gram-negative pathogens, including E coli, Shigella, and Salmonella. None of the other drugs listed would be appropriate. Many coliforms are now resistant to amoxicillin and ampicillin. Although trimethoprim is available as a single drug, resistance emerges rapidly during treatment unless it is used for urinary tract infections, in which high concentrations can be achieved. Vancomycin has no activity against gram-negative bacilli. The answer is B.

A nonindigenous person who develops onchocerciasis in an endemic region and receives drug treatment is likely to experience a severe reaction. Symptoms include headache, weakness, rash, muscle aches, hypotension, and peripheral edema. Which statement concerning this reaction is accurate? (A) Extensive fluid replacement is essential (B) Symptoms are more intense in indigenous adults than nonindigenous adults (C) The reaction is due to treatment with suramin (D) The reaction is due to killing of microfilariae (E) The symptoms are characteristic of treatment with diethylcarbamazine

2. The symptoms described are those of the so-called Mazzotti reaction. They are due to the killing by ivermectin of microfilariae, and their intensity correlates with skin microfilaria load and is not a drug toxicity. The reaction occurs more frequently and with greater severity in nonindigenous persons than in the indigenous inhabitants of endemic areas. The answer is D.

Questions 2 and 3. A 33-year-old man was seen in a clinic with a complaint of dysuria and urethral discharge of yellow pus. He had a painless clean-based ulcer on the penis and nontender enlargement of the regional lymph nodes. Gram stain of the urethral exudate showed gram-negative diplococci within polymorphonucleocytes. The patient informed the clinic staff that he was unemployed and had not eaten a meal for 2 d. 2. The most appropriate treatment of gonorrhea in this patient is (A) A single intramuscular dose of ceftriaxone (B) Amoxicillin orally for 7 d (C) Procaine penicillin G intramuscularly as a single dose plus oral probenecid (D) Meropenem orally for 7 d (E) Vancomycin intramuscularly as a single dose 3. Immunofluorescent microscopic examination of fluid expressed from the penile chancre of this patient revealed treponemes. Because he appears to be infected with Treponema pallidum, the best course of action would be to (A) Administer a single oral dose of fosfomycin (B) Give no other antibiotics because drug treatment of gonorrhea provides coverage for incubating syphilis (C) Inject intramuscular benzathine penicillin G (D) Treat with oral tetracycline for 7 d (E) Treat with vancomycin

2. Treatments of choice for gonorrhea include a single dose of ceftriaxone (intramuscularly). Because of the high incidence of beta-lactamase-producing gonococci, the use of penicillin G or amoxicillin is no longer appropriate for gonorrhea. Similarly, many strains of gonococci are resistant to tetracyclines. Alternative drugs (not listed) for gonorrhea include cefixime, azithromycin (see Chapter 44) or spectinomycin (see Chapter 45). The answer is A. 3. This patient with gonorrhea also has primary syphilis. The penile chancre, the enlarged nontender lymph nodes, and the microscopic identification of treponemes in fluid expressed from the lesion are essentials of diagnosis. Although a single dose of ceftriaxone may cure incubating syphilis, it cannot be relied on for treating primary syphilis. The most appropriate course of action in this patient is to administer a single intramuscular injection of 2.4 million units of benzathine penicillin G. For penicillin-allergic patients, oral doxycycline or tetracycline for 15 d (not 7 d) is effective in most cases (see Chapter 44). However, lack of compliance may be a problem with oral therapy. Fosfomycin and vancomycin have no significant activity against spirochetes. The answer is C

Which statement about pyrantel pamoate is accurate? (A) Acts as an antagonist at GABA receptors (B) Equivalent in efficacy to niclosamide in the treatment of tapeworm infections (C) Eradicates adult worms in the colon but not the eggs (D) Hepatotoxicity is dose-limiting (E) Synergistic with praziquantel in cestode infections

3. Pyrantel pamoate, an agonist at nicotinic receptors, is equivalent to albendazole and mebendazole in the treatment of common nematode infections. It acts on adult worms in the colon, but not on eggs. The drug causes only mild gastrointestinal side effects and is not hepatotoxic. It is not effective in the treatment of infections caused by cestodes or flukes. The answer is C.

A 31-year-old man has gonorrhea. He has no drug allergies, but a few years ago acute hemolysis followed use of an antimalarial drug. The physician is concerned that the patient has an accompanying urethritis caused by C trachomatis, although no cultures or enzyme tests have been performed. Which of the following drugs will be reliably effective against both gonococci and C trachomatis and safe to use in this patient? (A) Cefixime (B) Ciprofloxacin (C) Spectinomycin (D) Sulfamethoxazole-trimethoprim (E) None of the above

4. Cefixime in a single oral dose is effective in gonorrhea (Chapter 43), but it has no activity against organisms causing nongonococcal urethritis. Spectinomycin (Chapter 45) is active against most gonococci, but does not eradicate a urogenital chlamydial infection. Although ciprofloxacin might be effective in both gonorrhea and chlamydial urethritis, it is no longer recommended for treatment of gonorrhea in the United States, since resistance is now common. This patient could be treated by single oral doses of cefixime plus azithromycin (not listed). Sulfamethoxazole or TMP-SMZ would not be useful and may cause acute hemolysis in this patient. The answer is E.

A student studying medicine at a Caribbean university develops intestinal bilharziasis and oxamniquine is prescribed. Which statement about the proposed drug therapy is accurate? (A) Hospitalization is recommended if the patient has a history of seizure disorders (B) Oxamniquine is not effective in the late stages of the disease (C) Oxamniquine is safe to use in pregnancy (D) The drug is an antagonist at GABA receptors in trematodes (E) The drug is very effective in tapeworm infections

4. Oxamniquine may cause seizures, especially in persons with a history of convulsive disorders. Such persons should be hospitalized or treated with praziquantel. Oxamniquine is effective in all stages of disease caused by S mansoni, including advanced hepatosplenomegaly, and it has been used extensively for mass treatment. The drug is not effective in other schistosomal diseases, and it is contraindicated in pregnancy. The answer is A.

Which statement about the fluoroquinolones is accurate? (A) Antacids increase their oral bioavailability (B) Contraindicated in patients with hepatic dysfunction (C) Fluoroquinolones are drugs of choice in a 6-year-old child with a urinary tract (D) Gonococcal resistance to fluoroquinolones may involve changes in DNA gyrase (E) Modification of moxifloxacin dosage is required in patients when creatinine clearance is less than 50 mL/ min

5. Antacids can decrease oral bioavailability of fluoroquinolones. Neither hepatic or renal dysfunction is a contraindication to the use of fluoroquinolones. Most fluoroquinolones undergo renal elimination and dosage should be modified with creatinine clearance <50 mL/min. Moxifloxacin elimination occurs mainly via the liver. The fluoroquinolones should not be used to treat uncomplicated first-time urinary tract infections in children because of possible effects on cartilage development. Uncomplicated urinary tract infections in children are usually due to a strain of E coli that is sensitive to many other drugs, including beta-lactam antibiotics. The answer is D.

A 22-year-old man from South Korea has recently moved to Minnesota. He has symptoms of clonorchiasis (anorexia, upper abdominal pain, eosinophilia), presumably contracted in his homeland where the Oriental liver fluke is endemic. He also has symptoms of diphyllobothriasis (abdominal discomfort, diarrhea, megaloblastic anemia), probably caused by consumption of raw fish from lakes near the Canadian border. Which drug is most likely to be effective in the treatment of both clonorchiasis and diphyllobothriasis in this patient? (A) Albendazole (B) Ivermectin (C) Niclosamide (D) Praziquantel (E) Pyrantel pamoate

5. Praziquantel is a primary drug for treatment of infections caused by the Oriental liver fluke and by the fish tapeworm. Both types of infection are transmitted mainly via the consumption of raw fish. Niclosamide is also a primary drug for fish tapeworm infections, but it is not active against Clonorchis sinensis. Albendazole is not effective in fish tapeworm infections, but it is useful in the pork tapeworm larval stage (cysticercosis). Pyrantel pamoate is not active against cestodes or trematodes. The answer is D.

Which helminthic infection does not respond to treatment with praziquantel? (A) Hydatid disease (B) Opisthorchiasis (C) Paragonimiasis (D) Pork tapeworm infection (E) Schistosomiasis

6. In hydatid disease, praziquantel has marginal efficacy because it does not affect the inner germinal membrane of Echinococcus granulosus present in hydatid cysts. The answer is A.

A 40-year-old man complains of periodic bouts of diarrhea with lower abdominal cramping and intermittent rectal bleeding. Seen in the clinic, he appears well nourished, with blood pressure in the normal range. Examination reveals moderate abdominal pain and tenderness. His current medications are limited to loperamide for his diarrhea. Sigmoidoscopy reveals mucosal edema, friability, and some pus. Laboratory findings include mild anemia and decreased serum albumin. Microbiologic examination via stool cultures and mucosal biopsies do not reveal any evidence for bacterial, amebic, or cytomegalovirus involvement. The most appropriate drug to use in this patient is (A) Ampicillin (B) Doxycycline (C) Norfloxacin (D) Sulfasalazine (E) Trimethoprim-sulfamethoxazole

6. In the absence of any evidence pointing toward a definite microbial cause for the colitis in this patient, a drug that decreases inflammation is indicated. Sulfasalazine has significant anti-inflammatory action, and its oral use results in symptomatic improvement in 50-75% of patients suffering from ulcerative colitis. The drug is also used for its anti-inflammatory effects in rheumatoid arthritis. The answer is D.

Which of the following is a drug that is used in combination therapy for testicular carcinoma and is also associated with nephrotoxicity? (A) Bleomycin (B) Cisplatin (C) Etoposide (D) Leuprolide (E) Vinblastine

6. Nephrotoxicity is a characteristic toxicity of cisplatin. Renal toxicity can be reduced by slow intravenous infusion, maintenance of good hydration, and administration of mannitol to maximize urine flow. For testicular cancer, cisplatin is used in combination with etoposide and bleomycin. The answer is B.

Questions 6-8. A 52-year-old man (weight 70 kg) is brought to the hospital emergency department in a confused and delirious state. He has had an elevated temperature for more than 24 h, during which time he had complained of a severe headache and had suffered from nausea and vomiting. Lumbar puncture reveals an elevated opening pressure, and cerebrospinal fluid findings include elevated protein, decreased glucose, and increased neutrophils. Gram stain of a smear of cerebrospinal fluid reveals gram-positive diplococci, and a preliminary diagnosis is made of purulent meningitis. The microbiology report informs you that for approximately 15% of S pneumoniae isolates in the community, the minimal inhibitory concentration for penicillin G is 20 mcg/mL. 6. Treatment of this patient should be initiated immediately with intravenous administration of (A) Amoxicillin (B) Cephalexin (C) Ceftriaxone plus vancomycin (D) Nafcillin (E) Piperacillin 7. Resistance of pneumococci to penicillin G is due to (A) Alterations in porin structure (B) Beta-lactamase production (C) Changes in chemical structure of target penicillin-binding proteins (D) Changes in the d-Ala-d-Ala building block of peptidoglycan precursor (E) Decreased intracellular accumulation of penicillin G 8. If this patient had been 82-years-old and the Gram stain of the smear of cerebrospinal fluid had revealed gram-positive rods resembling diphtheroids, the antibiotic regimen for empiric treatment would include (A) Ampicillin (B) Cefoxitin (C) Ceftriaxone (D) Fosfomycin (E) Vancomyci

6. Pneumococcal isolates with a minimal inhibitory concentration for penicillin G of greater than 2 mcg/mL are highly resistant. Such strains are not killed by the concentrations of penicillin G or ampicillin that can be achieved in the cerebrospinal fluid. Nafcillin has minimal activity against penicillin-resistant pneumococci and piperacillin is mainly used for infections caused by gram-negative rods. Cefotaxime and ceftriaxone are the most active cephalosporins against penicillin-resistant pneumococci, and the addition of vancomycin is recommended in the case of highly resistant strains. The answer is C. 7. Pneumococcal resistance to penicillins is due to changes in the chemical structures of the target penicillin-binding proteins located in the bacterial cytoplasmic membrane. A similar mechanism underlies the resistance of staphylococci to methicillin (MRSA strains). A structural alteration in the d-Ala-d-Ala component of the pentapeptide side chains of peptidoglycans is the basis for a mechanism of resistance to vancomycin. The answer is C. 8. Diphtheroid-like gram-positive rods in the cerebrospinal fluid smear of an elderly patient are indicative of L monocytogenes. Listeria infections are more common in neonates, elderly patients, and those who have been treated with immunosuppressive agents. Treatment consists of ampicillin with or without an aminoglycoside such as gentamicin. Trimethoprim-sulfamethoxazole can also be used (see Chapter 46). The answer is A.

Questions 7-9. A 24-year-old woman comes to a clinic with complaints of dry cough, headache, fever, and malaise, which have lasted 3 or 4 d. She appears to have some respiratory difficulty, and chest examination reveals rales but no other obvious signs of pulmonary involvement. However, extensive patchy infiltrates are seen on chest x-ray film. Gram stain of expectorated sputum fails to reveal any bacterial pathogens. The patient mentions that a colleague at work had similar symptoms to those she is experiencing. The patient has no history of serious medical problems. She takes loratadine for allergies and supplementary iron tablets, and she drinks at least 6 cups of caffeinated coffee per day. The physician makes an initial diagnosis of communityacquired pneumonia. 7. Regarding the treatment of this patient, which of the following drugs is most suitable? (A) Ampicillin (B) Clindamycin (C) Doxycycline (D) Linezolid (E) Vancomycin 8. If this patient were to be treated with erythromycin, she should (A) Avoid exposure to sunlight (B) Avoid taking supplementary iron tablets (C) Decrease her intake of caffeinated beverages (D) Have her plasma urea nitrogen or creatinine checked before treatment (E) Temporarily stop taking loratadine 9. A 5-d course of treatment for community-acquired pneumonia would be effective in this patient with little risk of drug interactions if the drug prescribed were (A) Azithromycin (B) Clindamycin (C) Doxycycline (D) Erythromycin (E) Vancomycin

7. It is often difficult to establish a definite cause of communityacquired pneumonia (CAP). More than 80% of cases are caused by typical pathogens such as S pneumoniae, H influenzae, or M catarrhalis, and 15% are due to the nonzoonotic atypial pathogens such as Legionella species, Mycoplasma species, or C pneumoniae. Currently, monotherapy coverage of both typical and atypical pathogens in CAP is preferred to double-drug therapy. Preferred initial therapy includes a macrolide, doxycycline, or a quinolone active against respiratory pathogens (Chapter 46). Ampicillin, clindamycin, and vancomycin have low activity against atypical pathogens in CAP. The answer is C. 8. The inhibition of liver cytochrome P450 by erythromycin has led to serious drug interactions. Although erythromycin does not inhibit loratadine metabolism, it does inhibit the CYP1A2 form of cytochrome P450, which metabolizes methylxanthines.. Consequently, cardiac and/or CNS toxicity may occur with excessive ingestion of caffeine. Unlike the tetracyclines, the oral absorption of erythromycin is not affected by cations and the drug does not cause photosensitivity. Because erythromycin undergoes biliary excretion, there is little reason to assess renal function before treatment. The answer is C. 9. Azithromycin has a half-life of more than 70 h, which allows for once-daily dosing and a 5-d course of treatment for community-acquired pneumonia. Unlike other macrolides, azithromycin does not inhibit cytochrome P450 enzymes involved in drug metabolism. The answer is A.

Which drug enhances the actions of GABA in nematodes causing muscle paralysis? (A) Albendazole (B) Diethylcarbamazine (C) Ivermectin (D) Oxamniquine (E) Pyrantel pamoate

7. Ivermectin and piperazine (not listed) both cause muscle paralysis in nematodes by actions on GABA receptors. Pyrantel pamoate blocks nicotinic receptors relaxing nematode muscles. Diethylcarbamazine also cause muscle relaxation but the mechanism is unknown. Dizziness (no driving for 24 h) is a common adverse effect of oxamniquine. The answer is C.

A cancer cell that is resistant to the effects of both vincristine and methotrexate probably has developed the resistance as a result of which of the following mechanisms? (A) Changes in the properties of a target enzyme (B) Decreased activity of an activating enzyme (C) Increased expression of a P-glycoprotein transporter (D) Increased production of drug-trapping molecules (E) Increase in proteins that are involved in DNA repair

7. The P-glycoprotein family of transporters moves foreign molecules out of cells. Cancer cells acquire resistance to multiple drugs that act through different mechanisms by increasing the expressions of genes encoding these transporters. The answer is C.

Which adverse effect is most common with sulfonamides? (A) Fanconi's aminoaciduria syndrome (B) Hematuria (C) Kernicterus in the newborn (D) Neurologic dysfunction (E) Skin rash

7. The most common adverse effect of the sulfonamides is a skin rash caused by hypersensitivity. Neurologic dysfunction and hematuria occur less frequently. Sulfonamides are usually avoided in the third trimester of pregnancy or in neonates, so kernicterus is rare. Fanconi's syndrome is associated with the use of outdated tetracyclines. The answer is E.

Which parasite is susceptible to niclosamide? (A) Ascaris lumbricoides (roundworm) (B) Echinococcus granulosus (hydatid disease) (C) Fasciola hepatica (sheep liver fluke) (D) Necator americanus (hookworm) (E) Taenia solium (pork tapeworm)

8. Niclosamide is not active against nematodes or flukes with the exception of the large intestinal fluke. It is a co-drug of choice (with praziquantel) to treat common tapeworm infections and is usually effective in a single dose. The drug is minimally absorbed from the gastrointestinal tract and causes few side effects. The answer is E.

Which drug is effective in the treatment of nocardiosis and, in combination with pyrimethamine, is prophylactic against Pneumocystis jirovecii infections in AIDS patients? (A) Amoxicillin (B) Erythromycin (C) Levofloxacin (D) Sulfadiazine (E) Trimethoprim

8. Sulfadiazine and TMP-SMZ are drugs of choice in nocardiosis. In combination with pyrimethamine (an effective dihydrofolate reductase inhibitor in protozoa), sulfadiazine is effective in toxoplasmosis and is prophylactic against pneumocystis pneumonia in the AIDS patient. However, TMP-SMZ is more commonly used for the latter purpose. The answer is D.

A 10-year-old boy has uncomplicated pulmonary tuberculosis. After initial hospitalization, he is now being treated at home with isoniazid, rifampin, and ethambutol. Which statement about this case is accurate? (A) A baseline test of auditory function test is essential before drug treatment is initiated (B) His mother, who takes care of him, does not need INH prophylaxis (C) His 3-year-old sibling should receive INH prophylaxis (D) Polyarthralgia is a potential adverse effect of the drugs the boy is taking (E) The potential nephrotoxicity of the prescribed drugs warrants periodic assessment of renal function

A baseline test of ocular (not auditory) function may be useful before starting ethambutol. None of the drugs prescribed is associated with nephrotoxicity. Polyarthralgia is a common adverse effect of pyrazinamide that was not prescribed in this case. Periodic tests of liver function may be advisable in younger patients who are treated with INH plus rifampin, especially if higher doses of these drugs are used. Prophylaxis with INH is advisable for all household members and very close contacts of patients with active tuberculosis, especially young children. The answer is C.

Neuropathies are more likely to occur with this agent when it is used in patients with renal dysfunction. The drug may cause acute hemolysis in patients with glucose-6-phosphate dehydrogenase (G6PD) deficiency. (A) Chlorhexidine (B) Halazone (C) Methenamine (D) Metronidazole (E) Nitrofurantoin

Acute hemolytic reactions in G6PD deficiency occur with drugs that are oxidizing agents, including antimalarials, sulfonamides, and nitrofurans. Severe polyneuropathies may occur with nitrofurantoin, and they are more likely to occur in patients with renal dysfunction. The answer is E.

10. More than 90% of this drug is excreted in the urine in intact form. Because its urinary solubility is low, patients should be well hydrated to prevent nephrotoxicity. Which drug is described? (A) Acyclovir (B) Efavirenz (C) Indinavir (D) Trifluridine (E) Zidovudine

Acyclovir is eliminated in the urine by glomerular filtration and by active tubular secretion, which is inhibited by probenecid. Nephrotoxic effects, including hematuria and crystalluria, are enhanced in patients who are dehydrated or who have preexisting renal dysfunction. Adequate hydration is equally important in the case of indinavir because it causes nephrolithiasis. However, more than 80% of a dose of indinavir is eliminated via hepatic metabolism. Trifluridine is used topically to treat herpes keratoconjunctivitis. The answer is A.

A 3-year-old girl was referred to the genetic counselor by her pediatrician. She presents with short stature (height is 85 cm, -3 standard deviations) and appears to have loose skin on her neck. Cytogenetic testing reveals an XO karyotype. Which of the following drugs will allow her to achieve a higher adult height? (A) Adrenocorticotropin (ACTH) (B) Corticotropin-releasing hormone (CRH) (C) Growth hormone-releasing hormone (GHRH) (D) Gonadotropin-releasing hormone (GnRH) (E) Somatropin

Adrenocorticotropin (ACTH) is used diagnostically in suspected adrenal insufficiency. Corticotropin-releasing hormone (CRH) is used to distinguish Cushing's syndrome from ectopic ACTH secretion. GHRH is rarely used as treatment. Its main use is as a diagnostic tool. GnRH can be used to treat infertility. Somatropin, recombinant human GH, promotes growth in children with Turner's syndrome (an XO genetic genotype) or chronic renal failure. It also helps combat the AIDS-associated wasting syndrome. The answer is E.

Your 23-year-old female patient is pregnant and has gonorrhea. The medical history includes anaphylaxis following exposure to amoxicillin. The most appropriate drug to use is (A) Azithromycin (B) Cefixime (C) Ceftriaxone (D) Ciprofloxacin (E) Doxycycline

All of the listed drugs have been used for the treatment of gonorrhea. Cephalosporins should be avoided in patients with a history of severe hypersensitivity to penicillins, and fluoroquinolones (see Chapter 46) should be avoided in pregnancy. Tetracyclines including doxycycline have been used in the past for gonorrhea, but not as single doses, and they too should be avoided in pregnancy. The answer is A.

The primary reason for the use of drug combinations in the treatment of tuberculosis is to (A) Delay or prevent the emergence of resistance (B) Ensure patient compliance with the drug regimen (C) Increase antibacterial activity synergistically (D) Provide prophylaxis against other bacterial infections (E) Reduce the incidence of adverse effects

Although it is sometimes possible to achieve synergistic effects against mycobacteria with drug combinations, the primary reason for their use is to delay the emergence of resistance. The answer is A.

All of the following statements about the clinical uses of the aminoglycosides are accurate except (A) Effective in the treatment of infections caused by anaerobes such as Bacteroides fragilis (B) Gentamycin is used with ampicillin for synergistic effects in the treatment of enterococcal endocarditis (C) In the treatment of a hospital-acquired infection caused by Serratia marcescens, netilmicin is more effective than streptomycin (D) Often used with cephalosporins in the empiric treatment of life-threatening bacterial infections (E) Owing to their polar nature, aminoglycosides are not absorbed after oral administration

Aminoglycoside antibiotics act at the ribosomal level and their intracellular accumulation by bacteria is oxygen dependent. Anaerobic bacteria including B fragilis are innately resistant. The answer is A.

Regarding the mechanism of action of aminoglycosides, the drugs (A) Are bacteriostatic (B) Bind to the 50S ribosomal subunit (C) Cause misreading of the code on the mRNA template (D) Inhibit peptidyl transferase (E) Stabilize polysomes

Aminoglycosides are bactericidal inhibitors of protein synthesis binding to specific components of the 30S ribosomal subunit. Their actions include block of the formation of the initiation complex, miscoding, and polysomal breakup. Peptidyl transferase is inhibited by chloramphenicol, not aminoglycosides. The answer is C.

A 62-year-old woman presents with complaints of fatigue, sluggishness, and weight gain. She needs to nap several times a day, which is unusual for her. She has been taking T4 for the past 15 yr without significant problems regarding her energy level. Her recent history is significant for diagnosis of arrhythmia, and she is currently taking an antiarrhythmic drug. What is the most likely cause of her current condition? (A) Amiodarone (B) Lidocaine (C) Procainamide (D) Sotalol (E) Verapamil

Amiodarone is an iodine-containing antiarrhythmic drug with complex effects on the thyroid gland and thyroid hormones. One of its actions is to inhibit peripheral conversion of T4 to T3. Note that propranolol also reduces conversion of T4 to T3. Procainamide (class 1A), lidocaine (class 1B), sotalol (class III), and verapamil (class IV) are antiarrhythmics and have no effect on T4 conversion. The answer is A.

Which statement about antitubercular drugs is accurate? (A) Antimycobacterial actions of streptomycin involve inhibition of arabinosyltransferases (B) Cross-resistance of M tuberculosis to isoniazid and pyrazinamide is common (C) Ocular toxicity of ethambutol is prevented by thiamine (D) Pyrazinamide treatment should be discontinued immediately if hyperuricemia occurs (E) Resistance to ethambutol involves mutations in the emb gene

Arabinosyltransferase is inhibited by ethambutol (not streptomycin) and resistance involves alterations in the emb gene. Ocular adverse effects of ethambutol are dose-dependent and usually reversible when the drug is discontinued. Thiamine is not protective. There is minimal cross-resistance between pyrazinamide and other antimycobacterial drugs. Pyrazinamide uniformly causes hyperuricemia, but this is not a reason to halt therapy even though the drug may provoke gouty arthritis in susceptible persons. The answer is E.

Once-weekly administration of which of the following antibiotics has prophylactic activity against bacteremia caused by M avium complex in AIDS patients? (A) Acedapsone (B) Azithromycin (C) Clarithromycin (D) Kanamycin (E) Rifabutin

Because of its long elimination half-life (3-4 d), weekly administration of azithromycin has proved to be equivalent to daily administration of clarithromycin when used for prophylaxis against M avium complex in AIDS patients. Acedapsone is a repository form of dapsone used in leprosy. The answer is B.

Methenamine salts are used as urinary antiseptics. The reason they lack systemic antibacterial action is that they are (A) Converted to formaldehyde only at low pH (B) Metabolized rapidly by hepatic drug-metabolizing enzymes (C) More than 98% bound to plasma proteins (D) Not absorbed into the systemic circulation after oral ingestion (E) Substrates for active tubular secretion

Below pH 5.5, methenamine releases formaldehyde, which is antibacterial. This pH is achieved in the urine but nowhere else in the body. Ascorbic acid is sometimes given with methenamine salts to ensure a low urinary pH. The answer is A.

A 54-year-old obese patient with type 2 diabetes has a history of alcoholism. In this patient, metformin should either be avoided or used with extreme caution because the combination of metformin and ethanol increases the risk of which of the following? (A) A disulfiram-like reaction (B) Excessive weight gain (C) Hypoglycemia (D) Lactic acidosis (E) Serious hepatotoxicity

Biguanides, especially the older drug phenformin, have been associated with lactic acidosis. Thus, metformin should be avoided or used with extreme caution in patients with conditions that increase the risk of lactic acidosis, including acute ethanol ingestion. The answer is D.

A patient with multiple myeloma was started on bortezomib after 2 rounds of other combination chemotherapy did not have any effect. What is the mechanism of action of bortezomib? (A) Cross-linking of double-stranded DNA (B) Inhibition of DNA-dependent RNA synthesis (C) Interference with the activity of topoisomerases I (D) Inhibition of the 26S proteasome (E) Selective inhibition of DNA polymerases

Bortezomib is an inhibitor of the proteasome structure, whose normal function is to break down ubiquinated proteins. The answer is D.

A 27-year-old woman with amenorrhea, infertility, and galactorrhea was treated with a drug that successfully restored ovulation and menstruation. Before being given the drug, the woman was carefully questioned about previous mental health problems, which she did not have. She was advised to take the drug orally. Which of the following is most likely to be the drug that was used to treat this patient? (A) Bromocriptine (B) Desmopressin (C) Human gonadotropin hormone (D) Leuprolide (E) Octreotide

Bromocriptine, a dopamine receptor agonist, is used to treat the amenorrhea-galactorrhea syndrome, which is a consequence of hyperprolactinemia. Because of its central dopaminergic effects, the drug should not be used in patients with a history of schizophrenia or other forms of psychotic illness. The answer is A.

Which of the following cephalosporins has activity against gram-negative anaerobic pathogens like Bacteroides fragilis? A. Cefoxitin. B. Cefepime. C. Ceftriaxone. D. Cefazolin.

Correct answer = A. The cephamycins (cefoxitin and cefotetan) are the only cephalosporins with in vitro activity against anaerobic gram-negative pathogens. Cefepime, ceftriaxone, and cefazolin have no appreciable activity against Bacteroides fragilis.

A 26-year-old woman was treated for a suspected chlamydial infection at a neighborhood clinic. She was given a prescription for oral doxycycline to be taken for 14 d. Three weeks later, she returned to the clinic with a mucopurulent cervicitis. On questioning she admitted not having the prescription filled. The best course of action at this point would be to (A) Delay drug treatment until the infecting organism is identified (B) Rewrite the original prescription for oral doxycycline (C) Treat her in the clinic with a single oral dose of azithromycin (D) Treat her in the clinic with an intravenous dose of amoxicillin (E) Write a prescription for oral erythromycin for 10 d

Cervicitis or urethritis is often caused by C trachomatis. Such infections may develop slowly because of the long incubation period of chlamydial infection. Treatment with oral doxycycline for 14 d (as originally prescribed) would have eradicated C trachomatis and most other organisms commonly associated with nongonococcal cervicitis or urethritis. Given the limited compliance of this patient, the best course of action would be the administration (in the clinic) of a single oral dose of azithromycin. The answer is C.

A 4-year-old child is brought to the hospital after ingesting pills that a parent had used for bacterial dysentery when traveling outside the United States. The child has been vomiting for more than 24 h and has had diarrhea with green stools. She is now lethargic with an ashen color. Other signs and symptoms include hypothermia, hypotension, and abdominal distention. The drug most likely to be the cause of this problem is (A) Ampicillin (B) Azithromycin (C) Chloramphenicol (D) Doxycycline (E) Erythromycin

Chloramphenicol is commonly used outside the United States for treatment of bacillary dysentery. The drug causes a dose-dependent (reversible) suppression of erythropoiesis. Although the gray baby syndrome was initially described in neonates, a similar syndrome has occurred with overdosage of chloramphenicol in older children and adults, especially those with hepatic dysfunction. The answer is C.

Which statement about ciprofloxacin is accurate? (A) Antagonism occurs if used with dihydrofolate reductase inhibitors (B) Ciprofloxacin is active against MRSA strains of staphylococci (C) Most "first-time" urinary tract infections are resistant to ciprofloxacin (D) Organisms that commonly cause ear infections are highly resistant (E) Tendinitis may occur during treatment

Ciprofloxacin is commonly used for the treatment of urinary tract infections and is active against most strains of common causative agents of otitis media, including H influenzae and pneumococci. However, up to 50% of strains of MRSA are now resistant to ciprofloxacin. No clinical antagonism has been reported between fluoroquinolones and inhibitors of folic acid synthesis. Fluoroquinolones are not recommended for use in pregnancy or for children less than 10 years of age because they may damage growing cartilage. Tendonitis and tendon rupture are adverse effects of the fluoroquinolones. The answer is E.

Clarithromycin and erythromycin have very similar spectra of antimicrobial activity. The major advantage of clarithromycin is that it (A) Does not inhibit hepatic drug-metabolizing enzymes (B) Eradicates mycoplasmal infections in a single dose (C) Has greater activity against H pylori (D) Is active against methicillin-resistant strains of staphylococci (E) Is active against strains of streptococci that are resistant to erythromycin

Clarithromycin can be administered less frequently than erythromycin, but it is not effective in single doses against susceptible organisms. Organisms resistant to erythromycin, including pneumococci and methicillin-resistant staphylococci, are also resistant to other macrolides. Drug interactions have occurred with clarithromycin through its ability to inhibit cytochrome P450. Clarithromycin is more active than erythromycin against M avium complex, T gondii, and H pylori. The answer is C.

Which drug increases the hepatic metabolism of other drugs? (A) Clarithromycin (B) Erythromycin (C) Ketoconazole (D) Rifampin (E) Ritonavir

Clarithromycin, erythromycin, ketoconazole, and ritonavir inhibit the hepatic metabolism of various drugs. Rifampin, an inducer of liver microsomal drug-metabolizing enzymes can increase the metabolism of other drugs. The answer is D.

A 24-year-old female is diagnosed with genital herpes simplex virus infection. Which of the following agents is indicated for use in this diagnosis? A. Valacyclovir. B. Cidofovir. C. Ganciclovir. D. Zanamivir. E. Lamivudine.

Correct answer = A. Valacyclovir, famciclovir, penciclovir, and acyclovir are all indicated for herpes simplex virus infection. Cidofovir and ganciclovir are used for CMV retinitis. Zanamivir is indicated for influenza. Lamivudine is indicated for HIV and hepatitis B.

Which of the following corticosteroids is most appropriate to administer to a woman in preterm labor to accelerate fetal lung maturation? A. Betamethasone. B. Fludrocortisone. C. Hydrocortisone. D. Prednisone.

Correct answer = A. A corticosteroid with high glucocorticoid activity is needed to speed fetal lung maturation prior to delivery. Betamethasone has high glucocorticoid activity and is one of the recommended drugs in this context. Dexamethasone is the other. Fludrocortisone mainly has mineralocorticoid activity and would not be useful in this situation. Hydrocortisone has much lower glucocorticoid activity. Prednisone has a higher glucocorticoid activity than hydrocortisone, but the fetus is not able to convert it to prednisolone, the active form.

A patient with AIDS and a CD4 cell count of 100/μL has persistent fever and weight loss associated with invasive pulmonary disease due to M avium complex (MAC). Optimal management of this patient is to (A) Choose an antibiotic based on drug susceptibility of the cultured organism (B) Initiate a two-drug regimen of INH and pyrazinamide (C) Prescribe rifabutin because it prevents the development of MAC bacteremia (D) Start treatment with the combination of azithromycin, ethambutol, and rifabutin (E) Treat with trimethoprim-sulfamethoxazole

Combinations of antibiotics are essential for suppression of disease caused by M avium complex in the AIDS patient, and treatment should be started before culture results are available. Although rifabutin is prophylactic against MAC bacteremia when it is used as sole therapy in active disease, resistant strains of the organism emerge rapidly. MAC is much less susceptible than M tuberculosis to conventional antimycobacterial drugs. Currently, the optimum regimen consists of azithromycin (or clarithromycin) with ethambutol and rifabutin. The answer is D.

A 52-year-old postmenopausal patient has evidence of low bone mineral density. She and her physician are considering therapy with raloxifene or a combination of conjugated estrogens and medroxyprogesterone acetate. Which of the following patient characteristics is most likely to lead them to select raloxifene? (A) Previous hysterectomy (B) Recurrent vaginitis (C) Rheumatoid arthritis (D) Strong family history of breast cancer (E) Troublesome hot flushes

Conjugated estrogens and raloxifene both improve bone mineral density and protect against osteoporosis. The 2 advantages of raloxifene over full estrogen receptor agonists are that raloxifene has antagonist effects in breast tissue and lacks an agonistic effect in endometrium. If a patient's uterus was removed by surgery, the difference in the endometrial effect is moot. In patients with a strong family history of breast cancer, raloxifene may be a better choice than a full estrogen agonist because it will not further increase the woman's risk of breast cancer and may even lower her risk. The answer is D.

A 25-year-old male presents to the urgent care center with a painless sore on his genitals that started 1 to 2 weeks ago. He reports unprotected sex with a new partner about a month ago. A blood test confirms the patient has Treponema pallidum. Which of the following is the drug of choice for the treatment of this patient's infection as a single dose? A. Benzathine penicillin G. B. Ceftriaxone. C. Aztreonam. D. Vancomycin.

Correct answer = A. A single treatment with penicillin is curative for primary and secondary syphilis. No antibiotic resistance has been reported, and it remains the drug of choice unless the patient has a severe allergic reaction.

FL is a 64-year-old male about to undergo therapy for rhabdomyosarcoma. His chemotherapy includes ifosfamide. Which of the following is most appropriate to include in chemotherapy orders for this patient? A. IV hydration, mesna, and frequent urinalyses. B. Leucovorin and frequent urinalyses. C. Allopurinol and frequent urinalyses. D. IV hydration, prophylactic antibiotics, and frequent urinalyses.

Correct answer = A. A unique toxicity of ifosfamide is hemorrhagic cystitis. This bladder toxicity has been attributed to toxic metabolites of ifosfamide. Adequate hydration as well as IV injection of mesna (sodium 2-mercaptoethane sulfonate), which neutralizes the toxic metabolites, can minimize this problem. Frequent urinalyses to monitor for red blood cells should be ordered. Leucovorin is used with methotrexate or 5-FU (not ifosfamide). Allopurinol has a drug interaction with ifosfamide and is not an agent that prevents hemorrhagic cystitis. Prophylactic antibiotics are not needed.

Which of the following antibiotics exhibits a long postantibiotic effect that permits once-daily dosing? A. Gentamicin. B. Penicillin G. C. Vancomycin. D. Aztreonam.

Correct answer = A. Aminoglycosides, including gentamicin, possess a long post-antibiotic effect, especially when given as a high dose every 24 hours. Penicillin G, clindamycin, and vancomycin have a relatively short postantibiotic effect and require frequent dosing to maintain activity.

Which of the following antibiotic combinations is inappropriate based on antagonism at the same site of action? A. Clindamycin and erythromycin. B. Doxycycline and amoxicillin. C. Tigecycline and azithromycin. D. Ciprofloxacin and amoxicillin.

Correct answer = A. Clindamycin and erythromycin share the same site of action on the 50S ribosomal subunit and may result in antagonism, rendering both drugs ineffective. They also share cross-resistance.

The diagnosis of congenital adrenal hyperplasia (CAH) is confirmed in a child. This condition can be effectively treated by: A. Administering a glucocorticoid. B. Administering an androgen antagonist. C. Administering ketoconazole to decrease cortisol synthesis. D. Removing the adrenal gland surgically. E. Administering adrenocorticotropic hormone.

Correct answer = A. Congenital adrenal hyperplasia is seen in infancy and childhood. Because cortisol synthesis is decreased, feedback inhibition of adrenocorticotropic hormone (ACTH) formation and release is also decreased, resulting in enhanced ACTH formation. This in turn leads to increased levels of adrenal androgens and/or mineralocorticoids. The treatment is to administer a glucocorticoid, such as hydrocortisone (in infants) or prednisone, which would restore the feedback inhibition. The other options are inappropriate.

Cotrimoxazole would be expected to provide coverage for all of the following organisms except: A. Pseudomonas aeruginosa. B. Community-acquired MRSA. C. Nocardia asteroides. D. Stenotrophomonas maltophilia.

Correct answer = A. Cotrimoxazole is generally the drug of choice for answers C and D. It is also an excellent option for treatment of community-acquired MRSA skin and soft tissue infections.

DW is a patient with type 2 diabetes who has a blood glucose of 400 mg/dL today at his office visit. The physician would like to give some insulin to bring the glucose down before he leaves the office. Which of the following would lower the glucose in the quickest manner in DW? A. Insulin aspart. B. Insulin glargine. C. NPH insulin. D. Regular insulin.

Correct answer = A. Insulin aspart is a rapid-acting insulin that has an onset of action within 15 to 20 minutes. Insulin glargine is a long-acting insulin that is used for basal control. NPH insulin is an intermediate-acting insulin that is used for basal control. Although regular insulin can be used to bring the glucose down, its onset is not as quick as insulin aspart. The onset of regular insulin is about 30 to 60 minutes.

Which of the following medications used to treat river blindness targets chloride channels and can cause a Mazzotti reaction? A. Ivermectin. B. Praziquantel. C. Pyrantel pamoate. D. Albendazole.

Correct answer = A. Ivermectin targets the parasite's glutamate-gated chloride channel receptors. Chloride influx and hyperpolarization occur, resulting in paralysis of the worm.

An 18-year-old male is diagnosed with Chagas disease. Which medication would be the best for this patient? A. Nifurtimox. B. Suramin. C. Sodium stibogluconate. D. Metronidazole.

Correct answer = A. Nifurtimox is indicated for the treatment of American trypanosomiasis (Chagas disease) caused by T. cruzi. Suramin is used for the treatment of first-stage African trypanosomiasis due to T. brucei rhodesiense. Sodium stibogluconate is used for the treatment of leishmaniasis. Metronidazole is used for the treatment of amebiasis and giardiasis.

Which of the following is the primary method of β-lactam resistance with Streptococcus pneumoniae? A. Modification of target site. B. Decreased drug levels due to changes in permeability. C. Decreased drug levels due to an efflux pump. D. Enzymatic inactivation.

Correct answer = A. S. pneumoniae resistance to β-lactam antibiotics involves alteration in one or more of the major penicillin-binding proteins.

A 32-year-old male presents to an outpatient clinic with a 5-day history of productive cough, purulent sputum, and shortness of breath. He is diagnosed with communityacquired pneumonia (CAP). It is noted that this patient has a severe ampicillin allergy (anaphylaxis). Which of the following would be an acceptable treatment for this patient? A. Levofloxacin. B. Ciprofloxacin. C. Penicillin VK. D. Nitrofurantoin.

Correct answer = A. Streptococcus pneumoniae is a common cause of CAP, and the respiratory fluoroquinolones levofloxacin and moxifloxacin provide good coverage. Ciprofloxacin does not cover S. pneumoniae well and is a poor choice for treatment of CAP. Penicillin would be a poor choice due to allergy. Nitrofurantoin has no clinical utility for respiratory tract infections.

A 56-year-old female with diabetes presents for routine foot evaluation with her podiatrist. The patient complains of thickening of the nail of the right big toe and a change in color (yellow). The podiatrist diagnoses the patient with onychomycosis of the toenails. Which of the following is the most appropriate choice for treating this infection? A. Terbinafine. B. Micafungin. C. Itraconazole. D. Griseofulvin.

Correct answer = A. Terbinafine is better tolerated, requires a shorter duration of therapy, and is more effective than either itraconazole or griseofulvin. Micafungin is not active for this type of infection.

Which of the following medications inhibits the phosphorylation of adenosine diphosphate? A. Albendazole. B. Mebendazole. C. Niclosamide. D. Praziquantel.

Correct answer = C. Niclosamide inhibits the parasite's mitochondrial phosphorylation of adenosine diphosphate (ADP), which produces usable energy in the form of adenosine triphosphate (ATP).

A 72-year-old male is admitted to the hospital from a nursing home with severe pneumonia. He was recently discharged from the hospital 1 week ago after open heart surgery. The patient has no known allergies. Which of the following regimens is most appropriate for empiric coverage of methicillinresistant Staphylococcus aureus and Pseudomonas aeruginosa in this patient? A. Vancomycin + cefepime + ciprofloxacin. B. Vancomycin + cefazolin + ciprofloxacin. C. Telavancin + cefepime + ciprofloxacin. D. Daptomycin + cefepime + ciprofloxacin.

Correct answer = A. Vancomycin provides adequate coverage against MRSA, and cefepime and ciprofloxacin provide adequate empiric coverage of Pseudomonas. B is incorrect because cefazolin does not have activity against Pseudomonas. C is incorrect because telavancin should be avoided if possible with drugs that prolong the QTc interval, in this case ciprofloxacin. Daptomycin is inactivated by pulmonary surfactant and should not be used for pneumonia.

A 55-year-old female presents to the hospital with shortness of breath, fever, and malaise. She has a history of breast cancer, which was diagnosed 3 months ago, and has been treated with chemotherapy. Her chest x-ray shows possible pneumonia, and respiratory cultures are positive for Aspergillus fumigatus. Which of the following is the MOST appropriate choice for treatment? A. Voriconazole. B. Fluconazole. C. Flucytosine. D. Ketoconazole.

Correct answer = A. Voriconazole is the drug of choice for aspergillosis. Studies have found it to be superior to other regimens including amphotericin B. Fluconazole, flucytosine, and ketoconazole do not have reliable in vitro activity and are therefore not recommended.

All of the following factors influence the penetration and concentration of an antibacterial agent in the cerebrospinal fluid except: A. Lipid solubility of the drug. B. Minimum inhibitory concentration of the drug. C. Protein binding of the drug. D. Molecular weight of the drug

Correct answer = B. Although the minimum inhibitory concentration will impact the effectiveness of the drug against a given bacteria, it does not affect the ability of a drug to penetrate into the brain. The lipid solubility, protein binding, and molecular weight all determine the likelihood of a drug to penetrate the blood-brain barrier and concentrate in the brain.

Which of the following antifungal agents is MOST likely to cause renal insufficiency? A. Fluconazole. B. Amphotericin B. C. Itraconazole. D. Posaconazole.

Correct answer = B. Amphotericin B is the best choice since nephrotoxicity is commonly associated with this medication. Although the dose of fluconazole must be adjusted for renal insufficiency, it is not associated with causing nephrotoxicity. Itraconazole and posaconazole are metabolized by the liver and are not associated with nephrotoxicity.

A 30-year-old pregnant female has cellulitis caused by MRSA. Which of the following antibiotics would be the most appropriate option for outpatient therapy? A. Doxycycline. B. Clindamycin. C. Quinupristin/dalfopristin. D. Tigecycline.

Correct answer = B. Clindamycin is the safest option for the treatment of MRSA in a pregnant patient. Doxycycline and tigecycline can cross the placenta and can cause harm to the fetus. Moreover, quinupristin/dalfopristin and tigecycline are only available intravenously and would not be appropriate for home antibiotic therapy for the given indication.

A 40-year-old female is undergoing infertility treatments. Which of the following drugs might be included in her treatment regimen? A. Cabergoline. B. Follitropin. C. Methimazole. D. Vasopressin.

Correct answer = B. Follitropin is a recombinant version of FSH that causes ovarian follicular growth and maturation. Cabergoline is a dopamine agonist that is used for hyperprolactinemia. Methimazole is the treatment of choice for hyperthyroidism. Vasopressin is an antidiuretic hormone.

Osteoporosis is a major adverse effect caused by the glucocorticoids. It is due to their ability to: A. Increase the excretion of calcium. B. Inhibit absorption of calcium. C. Stimulate the hypothalamic-pituitary-adrenal axis. D. Decrease production of prostaglandins. E. Decrease collagen synthesis.

Correct answer = B. Glucocorticoid-induced osteoporosis is attributed to inhibition of calcium absorption as well as bone formation. Increased intake of calcium plus vitamin D, bisphosphonates, or other drugs that are effective in this condition is indicated. Glucocorticoids suppress rather than stimulate the hypothalamic-pituitary-adrenal axis. The decreased production of prostaglandins does not play a role in bone formation.

Which of the following drugs is correctly matched with the appropriate adverse effect? A. Levofloxacin—hyperkalemia. B. Nitrofurantoin—pulmonary fibrosis. C. Cotrimoxazole—hepatic encephalopathy. D. Methenamine—nystagmus.

Correct answer = B. Hyperkalemia may be caused by cotrimoxazole, not fluoroquinolones. Hepatic encephalopathy may be related to therapy with methenamine in patients with hepatic insufficiency. Nystagmus is not associated with methenamine therapy.

Which of the following drugs for diabetes would be LEAST likely to cause weight gain? A. Glimepiride. B. Liraglutide. C. Pioglitazone. D. Repaglinide. E. Insulin glulisine.

Correct answer = B. Incretin mimetics are usually associated with weight loss due to their ability to enhance satiety. All of the other agents are associated with weight gain.

Which of the following statements is correct regarding insulin glargine? A. It is primarily used to control postprandial hyperglycemia. B. It is a "peakless" insulin. C. The prolonged duration of activity is due to slow dissociation from albumin. D. It should not be used in a regimen with insulin lispro or glulisine. E. It may be administered intravenously in emergency cases.

Correct answer = B. Insulin glargine has a relatively flat, prolonged hypoglycemic effect. Because of this it is used for basal glucose control, not postprandial. The prolonged duration is due to its low pH, which leads to precipitation at the injection site and resultant extended action. Insulin glargine is often used for basal control in a regimen where insulin lispro, glulisine, or aspart are used for mealtime glucose control. [Note: Glargine should not be combined with other insulins in the same syringe, as it may alter the pharmacodynamic properties of the medication.]

After the acute infection, which of the following medications is given to treat the asymptomatic colonization state of E. histolytica? A. Chloroquine. B. Iodoquinol. C. Metronidazole. D. Primaquine.

Correct answer = B. Iodoquinol, diloxanide furoate, and paromomycin are luminal amebicides that are usually administered with mixed or systemic amebicides to treat the asymptomatic colonization state. Chloroquine is a systemic amebicide and an antimalarial. Metronidazole is a mixed amebicide. Primaquine is an antimalarial.

Which of the following is the treatment of choice for hypothyroidism? A. Iodide. B. Levothyroxine. C. Liothyronine. D. Liotrix. E. Propylthiouracil

Correct answer = B. Levothyroxine is preferred due to its long half-life and better tolerability. Liothyronine (T3) and liotrix (T3/T4) are not as well tolerated. Iodide and propylthiouracil are used in the treatment of hyperthyroidism

Which of the following statements is characteristic of metformin? A. Metformin is inappropriate for initial management of type 2 diabetes. B. Metformin decreases hepatic glucose production. C. Metformin undergoes significant metabolism via the cytochrome P450 system. D. Metformin should not be combined with sulfonylureas or insulin. E. Weight gain is a common adverse effect.

Correct answer = B. Metformin works by inhibiting hepatic gluconeogenesis. It is the preferred initial agent for management of type 2 diabetes. Metformin is not metabolized. It may be combined with sulfonylureas, insulin, or TZDs. Unlike the sulfonylureas and insulin, weight gain is not an adverse effect, and some patients actually lose weight due to GI side effects.

Which of the following agents is available as an oral therapy for the treatment of visceral leishmaniasis? A. Artemether/lumefantrine. B. Miltefosine. C. Nitazoxanide. D. Tinidazole.

Correct answer = B. Miltefosine is the only oral agent available for the treatment of visceral leishmaniasis. All the other drugs are orally administered, but artemether/lumefantrine is used for the treatment of malaria, nitazoxanide is used for the treatment of giardiasis or cryptosporidiosis, and tinidazole is effective for amebiasis or giardiasis.

A 75-year-old man with chronic obstructive pulmonary disease is diagnosed with suspected influenza based on his complaints of flu-like symptoms that began 24 hours ago. Which of the following agents is most appropriate to initiate for the treatment of influenza? A. Ribavirin. B. Oseltamivir. C. Zanamivir. D. Rimantadine. E. Amantadine.

Correct answer = B. Oseltamivir is the best choice since it is administered orally and not associated with resistance. Zanamivir is administered via inhalation and is not recommended for patients with underlying COPD. High rates of resistance have developed to adamantanes (amantadine, rimantadine), and these drugs are infrequently indicated. Ribavirin is not indicated for treatment of influenza.

The appearance of a facial rash with cetuximab is associated with which of the following? A. A negative response to therapy. B. A positive response to therapy. C. A drug allergy. D. An infusion reaction.

Correct answer = B. Patients undergoing therapy with an EGFR inhibitor such as cetuximab often develop an acnelike rash on the face, chest, upper back, and arms. The appearance of such a rash has been correlated with an increased response as compared to patients who do not experience a rash during therapy.

A 45-year-old male patient is being treated with ABVD chemotherapy for Hodgkin lymphoma. He presents for cycle 4 of a planned 6 cycles with a new-onset cough. He states it started a week ago and he also feels like he has a little trouble catching his breath. Which drug in the ABVD regimen is the most likely cause of his pulmonary toxicity? A. Doxorubicin (Adriamycin). B. Bleomycin. C. Vinblastine. D. Dacarbazine.

Correct answer = B. Pulmonary toxicity is the most serious adverse effect of bleomycin, progressing from rales, cough, and infiltrate to potentially fatal fibrosis. The pulmonary fibrosis that is caused by bleomycin is often referred as "bleomycin lung."

A 42-year-old male HIV patient was recently diagnosed with active tuberculosis. Currently, he is on a stable HIV regimen consisting of two protease inhibitors and two nucleoside reverse transcriptase inhibitors (NRTIs). What is the most appropriate regimen to use for treatment of his tuberculosis? A. Rifampin + isoniazid + pyrazinamide + ethambutol. B. Rifabutin + isoniazid + pyrazinamide + ethambutol. C. Rifapentine + isoniazid + pyrazinamide + ethambutol. D. Rifampin + moxifloxacin + pyrazinamide + ethambutol. E. Amikacin + moxifloxacin + cycloserine + streptomycin.

Correct answer = B. Rifabutin is recommended in place of rifampin in patients coinfected with HIV, since it is a less potent inducer of CYP enzymes than rifampin. However, rifabutin is a CYP3A4 substrate and "bidirectional" interactions may result. Other medications, such as the protease inhibitors, may affect the concentration of rifabutin, requiring a dose adjustment. Answer E is incorrect as these are not first-line agents.

A patient is about to undergo three cycles of chemotherapy prior to surgery for bladder cancer. Which of the following best describes chemotherapy in this setting? A. Adjuvant. B. Neoadjuvant. C. Palliative. D. Maintenance.

Correct answer = B. Since the chemotherapy is being given before the surgery, it is considered neoadjuvant. Chemotherapy is indicated when neoplasms are disseminated and are not amenable to surgery (palliative). Chemotherapy is also used as a supplemental treatment to attack micrometastases following surgery and radiation treatment, in which case it is called adjuvant chemotherapy. Chemotherapy given prior to the surgical procedure in an attempt to shrink the cancer is referred to as neoadjuvant chemotherapy, and chemotherapy given in lower doses to assist in prolonging a remission is known as maintenance chemotherapy.

Which of the following adverse drug reactions precludes a patient from being rechallenged with that drug in the future? A. Itching/rash from penicillin. B. Stevens-Johnson syndrome from sulfamethoxazole-trimethoprim. C. Gastrointestinal (GI) upset from clarithromycin. D. Clostridium difficile superinfection from moxifloxacin.

Correct answer = B. Stevens-Johnson syndrome is a severe idiosyncratic reaction that can be life threatening, and these patients should never be rechallenged with the offending agent. Itching/rash is a commonly reported reaction in patients receiving penicillins but is not life threatening. A patient may be rechallenged if the benefits outweigh the risk (for example, pregnant patient with syphilis) or the patient could be exposed through a desensitization procedure. GI upset is a common side effect of clarithromycin but is not due to an allergic reaction. Moxifloxacin is a broadspectrum antibiotic that can inhibit the normal flora of the GI tract, increasing the risk for the development of superinfections like C. difficile. This is not an allergic reaction, and the patient can be rechallenged; however, the patient might be at risk for developing C. difficile infection again.

Which of the following drugs requires patients to take a multivitamin while on the medication? A. Phentermine. B. Phentermine/topiramate. C. Orlistat. D. Diethylpropion. E. Lorcaserin.

Correct answer = C. Orlistat interferes with the absorption of fat-soluble vitamins and β-carotene. It is important that individuals take the multivitamin 2 hours before or after administration of the orlistat.

Which of the following patients would most likely have suppression of the HPA axis and require a slow taper of corticosteroid therapy? A. A patient taking 40 mg of prednisone daily for 7 days to treat an asthma exacerbation. B. A patient taking 10 mg of prednisone daily for 3 months for rheumatoid arthritis. C. A patient using mometasone nasal spray daily for 6 months for allergic rhinitis. D. A patient receiving an intra-articular injection of methylprednisolone for osteoarthritis.

Correct answer = B. Suppression of the HPA axis usually occurs with higher doses of corticosteroids when used for a duration of 2 weeks or more. Although the dose of prednisone is higher in the asthma patient, the duration of therapy is short, so the risk of HPA axis suppression is lower. The risk of HPA axis suppression is low with topical therapies like intranasal mometasone and with one-time joint injections

Children younger than 8 years of age should not receive tetracyclines because these agents: A. Cause rupture of tendons. B. Deposit in tissues undergoing calcification. C. Do not cross into the cerebrospinal fluid. D. Can cause aplastic anemia.

Correct answer = B. Tetracyclines are contraindicated in this age group because they are deposited in tissues undergoing calcification, such as teeth and bone, and can stunt growth. Ciprofloxacin can interfere in cartilage formation and cause rupture of tendons and is also contraindicated in children, but it is a fluoroquinolone. Tetracyclines can cross into the cerebrospinal fluid. They do not cause aplastic anemia, a property usually associated with chloramphenicol.

Which of the following zones of the adrenal gland is correctly paired with the type of substance it secretes? A. Adrenal medulla—corticotropin. B. Zona fasciculata—cortisol. C. Zona glomerulosa—androgens. D. Zona reticularis—catecholamines.

Correct answer = B. The adrenal medulla secretes catecholamines. Corticotropin is secreted by the anterior pituitary. The zona glomerulosa secretes aldosterone, and the zona reticularis secretes androgens.

A 27-year-old recently married female is asking about treatment options for her obesity. She recently stopped taking her birth control medications, as she felt these were contributing to her weight gain. Which of the following medications should be avoided in this patient? A. Phentermine. B. Phentermine/topiramate. C. Orlistat. D. Diethylpropion. E. Lorcaserin.

Correct answer = B. The topiramate component of this medication is contraindicated in pregnancy. Since this patient stopped her birth control, she is at risk of becoming pregnant and her fetus is at risk of developing birth defects if she is taking this medication.

Which of the following antifungal agents should be avoided in patients with evidence of ventricular dysfunction? A. Micafungin. B. Itraconazole. C. Terbinafine. D. Posaconazole.

Correct answer = B. There is a black box warning that warns against the use of itraconazole in patients with evidence of ventricular dysfunction, including patients with heart failure.

A patient with Addison disease is being treated with hydrocortisone but is still having problems with dehydration and hyponatremia. Which of the following drugs would be best to add to the patient's therapy? A. Dexamethasone. B. Fludrocortisone. C. Prednisone. D. Triamcinolone.

Correct answer = B. To combat dehydration and hyponatremia, a corticosteroid with high mineralocorticoid activity is needed. Fludrocortisone has the greatest mineralocorticoid activity of the agents provided. The other drugs have little or no mineralocorticoid activity.

A 23-year-old male presents with acute appendicitis that ruptures shortly after admission. He is taken to the operating room for surgery, and postsurgical cultures reveal Escherichia coli and Bacteroides fragilis, susceptibilities pending. Which of the following provides adequate empiric coverage of these two pathogens? A. Cefepime. B. Piperacillin/tazobactam. C. Aztreonam. D. Ceftaroline.

Correct answer = B. While all of these agents cover most strains of E. coli, piperacillin/tazobactam is the only drug on this list that provides coverage against Bacteroides species.

In which of the following cases would it be appropriate to use telavancin? A. A 29-year-old pregnant female with ventilatorassociated pneumonia. B. A 76-year-old male with hospital-acquired pneumonia also receiving amiodarone for atrial fibrillation. C. A 36-year-old male with cellulitis and abscess growing MRSA. D. A 72-year-old female with a diabetic foot infection growing MRSA who has moderate renal dysfunction.

Correct answer = C. A is not a good option due to the potential of telavancin harming the fetus. Option B is not a good choice because the patient is on amiodarone, and telavancin can cause QTc prolongation. Option D is not an appropriate choice because the patient has baseline renal dysfunction and telavancin should be avoided unless benefit outweighs the risk. Option C is the best choice in this case since it is approved for skin and skin structure infections, and the patient has no apparent contraindication.

A 68-year-old male presents from a nursing home with fever, increased urinary frequency and urgency, and mental status changes. He has a penicillin allergy of anaphylaxis. Which of the following β-lactams is the most appropriate choice for gram-negative coverage of this patient's urinary tract infection? A. Cefepime. B. Ertapenem. C. Aztreonam. D. Ceftaroline.

Correct answer = C. Based on the severity of the allergic reaction, aztreonam is the choice of all the β-lactams. Although cross-reactivity with cephalosporins and carbapenems is low, the risk rarely outweighs the benefit in these cases.

A 20-year-old female presents to the emergency room with headache, stiff neck, and fever for 2 days and is diagnosed with meningitis. Which of the following agents is the best choice for the treatment of meningitis in this patient? A. Cefazolin. B. Cefdinir. C. Cefotaxime. D. Cefuroxime axetil.

Correct answer = C. Cefotaxime is the only drug on this list with adequate CSF penetration to treat meningitis. Cefdinir and cefuroxime axetil are only available orally, and cefazolin CSF penetration and spectrum of coverage against S. pneumoniae are not likely adequate to treat meningitis.

A patient is being discharged from the hospital on a 3-week course of clindamycin. Which of the following potential adverse effects should be discussed with her? A. Hyperbilirubinemia. B. Nephrotoxicity. C. Clostridium difficile diarrhea. D. Pseudotumor cerebri.

Correct answer = C. Clindamycin, among other antibiotics, is associated with the development of C. difficile and pseudomembranous colitis due to disruption of normal gut flora, particularly with prolonged therapy. Hyperbilirubinemia is associated with quinupristin/dalfopristin, nephrotoxicity is associated with aminoglycosides, and pseudotumor cerebri can occur with tetracyclines.

Which of the following is correct regarding clofazimine in the treatment of leprosy? A. Clofazimine should not be used in patients with a deficiency in glucose-6-phosphate dehydrogenase (G6PD). B. Peripheral neuropathy is one of the most common adverse effects seen with the drug. C. Clofazimine may cause skin discoloration over time. D. The risk of erythema nodosum leprosum is increased in patients given clofazimine.

Correct answer = C. Clofazimine is a phenazine dye and will cause bronzing (the skin pigment color will change color, from pink to brownish-black), especially in fair-skinned patients. This occurs in a majority of patients, and generally is not considered harmful but may take several months to years to fade after discontinuing the medication.

A child with severe asthma is being treated with high doses of inhaled corticosteroids. Which of the following adverse effects is of particular concern? A. Hypoglycemia. B. Hirsutism. C. Growth suppression. D. Cushing syndrome. E. Cataract formation.

Correct answer = C. Corticosteroids may retard bone growth. Chronic treatment with the medication therefore may lead to growth suppression, so linear growth should be monitored periodically. Hyperglycemia, not hypoglycemia, is a possible adverse effect. Hirsutism, Cushing syndrome, and cataract formation are unlikely with the dose that the child would receive by inhalation.

Corticosteroids are useful in the treatment of all of the following disorders except: A. Addison disease. B. Allergic rhinitis. C. Cushing syndrome. D. Inflammatory bowel disease. E. Rheumatoid arthritis.

Correct answer = C. Cushing syndrome is an excess secretion of glucocorticoids. Dexamethasone may be used in the diagnosis of Cushing syndrome, but not its treatment. Treatment of Cushing syndrome usually consists of surgery or suppression of glucocorticoid production with ketoconazole or other agents. Corticosteroids are used frequently in the management of the other disorders listed.

A 45-year-old male presented to the hospital 3 days ago with severe cellulitis and a large abscess on his left leg. Incision and drainage were performed on the abscess, and cultures revealed methicillin-resistant Staphylococcus aureus. Which of the following would be the most appropriate treatment option for oncedaily outpatient intravenous therapy? A. Ertapenem. B. Ceftaroline. C. Daptomycin. D. Piperacillin/tazobactam.

Correct answer = C. Daptomycin is approved for skin and skin structure infections caused by MRSA and is given once daily. A and D are incorrect because they do not cover MRSA. Ceftaroline covers MRSA, but it must be given twice daily.

A 58-year-old male with a history of hepatitis C, cirrhosis, and ascites presents with spontaneous bacterial peritonitis. Which of the following antibiotics requires close monitoring and dosing adjustment in this patient given his liver disease? A. Penicillin G. B. Tobramycin. C. Erythromycin. D. Vancomycin.

Correct answer = C. Erythromycin is metabolized by the liver and should be used with caution in patients with hepatic impairment. Penicillin G, tobramycin, and vancomycin are primarily eliminated by the kidneys.

A 53-year-old woman has severe vasomotor symptoms (hot flushes) associated with menopause. She has no pertinent past medical or surgical history. Which of the following would be most appropriate for her symptoms? A. Conjugated estrogens vaginal cream. B. Estradiol transdermal patch. C. Oral estradiol and medroxyprogesterone acetate. D. Injectable medroxyprogesterone acetate.

Correct answer = C. Estrogen vaginal cream only treats vaginal symptoms of menopause such as vaginal atrophy and does not treat hot flushes. Since this patient has an intact uterus, a progestin such as medroxyprogesterone needs to be used along with the estrogen to prevent the development of endometrial hyperplasia. Unopposed estrogen (for example, the estradiol transdermal patch) should not be used. Injectable medroxyprogesterone acetate is used for contraception.

All of the following adverse effects commonly occur with glucocorticoid therapy except: A. Glaucoma. B. Increased risk of infection. C. Hypotension. D. Emotional disturbances. E. Peripheral edema.

Correct answer = C. Glucocorticoid therapy may cause hypertension, not hypotension. All the other adverse effects are associated with the use of glucocorticoids.

Which of the following agents is considered a narrowspectrum antibiotic? A. Ceftriaxone. B. Ciprofloxacin. C. Isoniazid. D. Imipenem.

Correct answer = C. Isoniazid is only active against Mycobacterium tuberculosis, while ceftriaxone, ciprofloxacin, and imipenem are considered broad spectrum due to their activity against multiple types of bacteria and risk for developing a superinfection.

26-year-old female is using injectable medroxyprogesterone acetate as a method of contraception. Which of the following adverse effects is a concern if she wishes to use this therapy long-term? A. Hyperkalemia. B. Male pattern baldness. C. Osteoporosis. D. Weight loss

Correct answer = C. Medroxyprogesterone acetate may contribute to bone loss and predispose patients to osteoporosis and/or fractures. Therefore, the drug should not be continued for more than 2 years if possible. The drug often causes weight gain, not weight loss. The other adverse effects are not associated with medroxyprogesterone.

Which of the following is the most appropriate initial oral agent for management of type 2 diabetes in patients with no other comorbid conditions? A. Glipizide. B. Insulin. C. Metformin. D. Pioglitazone.

Correct answer = C. Metformin is the preferred initial agent for management of type 2 diabetes. See Figure 25.14.

KD is a 69-year-old male with type 2 diabetes and advanced chronic kidney disease. Which of the following diabetes medications is contraindicated in this patient? A. Glipizide. B. Insulin lispro. C. Metformin. D. Saxagliptin.

Correct answer = C. Metformin should not be used in patients with kidney disease due to the possibility of lactic acidosis. Glipizide can be used safely in patients with CrCl as low as 10 mL/min. Insulin is not contraindicated in renal dysfunction, although the dosage may need to be adjusted. While the dose of the DPP-4 inhibitor saxagliptin may need to be reduced in renal dysfunction, it is not contraindicated.

A 22-year-old female presents with a 2-day history of dysuria with increased urinary frequency and urgency. A urine culture and urinalysis are done. She is diagnosed with a urinary tract infection (UTI) caused by E. coli. All of the following would be considered appropriate therapy for this patient except: A. Levofloxacin. B. Cotrimoxazole. C. Moxifloxacin. D. Nitrofurantoin.

Correct answer = C. Moxifloxacin does not concentrate in the urine and would be ineffective for treatment of a UTI. All other answers are viable alternatives, and the resistance profile for the E. coli can be utilized to direct therapy.

A 45-year-old female presents seeking treatment for weight loss. She has tried several fad diets in the past with very little success. She exercises twice weekly at the gym for 30 minutes and tries to watch what she eats. Her BMI is 31 and she has diabetes and uncontrolled hypertension. Which of the following medications would be most appropriate to treat her obesity? A. Phentermine. B. Phentermine/topiramate. C. Orlistat. D. Diethylpropion.

Correct answer = C. Orlistat is the only medication of those listed that does not increase heart rate and blood pressure. Since this patient's blood pressure is currently uncontrolled, choosing a drug that does not affect blood pressure would be best at this time.

A 70-year-old woman is being treated with raloxifene for osteoporosis. Which of the following is a concern with this therapy? A. Breast cancer. B. Endometrial cancer. C. Venous thrombosis. D. Hypercholesterolemia.

Correct answer = C. Raloxifene can increase the risk of venous thromboembolism. Unlike estrogen and tamoxifen, raloxifene does not result in an increased incidence of endometrial cancer. Raloxifene lowers the risk of breast cancer in high-risk women, and it also lowers LDL cholesterol

Which of the following classes of oral diabetes drugs is paired most appropriately with its primary mechanism of action? A. DPP-4 inhibitor—inhibits breakdown of complex carbohydrates. B. Glinide—increases insulin sensitivity. C. Sulfonylurea—increases insulin secretion. D. Thiazolidinedione—decreases hepatic gluconeogenesis.

Correct answer = C. Sulfonylureas work primarily by increasing insulin secretion through stimulation of the β cells of the pancreas. DPP-4 inhibitors work by inhibiting breakdown of incretins, thereby increasing postprandial insulin secretion, decreasing postprandial glucagon, etc. Glinides work primarily by increasing insulin secretion. TZDs work primarily by increasing insulin sensitivity.

A 30-year-old male patient with human immunodeficiency virus infection is being treated with a HAART (highly active antiretroviral therapy) regimen. Four weeks after initiating therapy, he presents to the emergency department complaining of fever, rash, and gastrointestinal upset. Which one of the following drugs is most likely the cause of his symptoms? A. Zidovudine. B. Nelfinavir. C. Abacavir. D. Efavirenz. E. Darunavir.

Correct answer = C. The abacavir hypersensitivity reaction is characterized by fever, rash, and gastrointestinal upset. The patient must stop therapy and not be rechallenged.

A group of college students are traveling to a chloroquine-resistant malaria area for a mission trip. Which of the following medications can be used for both prevention and treatment of malaria in these students? A. Pyrimethamine. B. Artemisinin. C. Atovaquone-proguanil. D. Melarsoprol.

Correct answer = C. The combination of atovaquone- proguanil has been used for both prevention and treatment of malaria in chloroquine-resistant areas. Pyrimethamine is not recommended for prophylaxis of malaria. Artemisinin and its derivatives are not used for prophylaxis, only treatment of malaria. Melarsoprol is used for the treatment of African sleeping sickness.

Which of the following is the most appropriate oral contraceptive for a patient with moderate acne? A. Ethinyl estradiol/levonorgestrel. B. Ethinyl estradiol/norethindrone acetate. C. Ethinyl estradiol/norgestimate. D. Ulipristal.

Correct answer = C. The progestins levonorgestrel and norethindrone acetate may have androgenic activity and contribute to acne. Norgestimate has less androgenic activity and is preferred for this patient. Ulipristal is an emergency contraceptive and should not be used as a regular method of contraception.

A 48-year-old immigrant from Mexico presents with seizures and other neurologic symptoms. Eggs of T. solium are found upon examination of a stool specimen. A magnetic resonance image of the brain shows many cysts, some of which are calcified. Which one of the following drugs would be of benefit to this individual? A. Ivermectin. B. Pyrantel pamoate. C. Albendazole. D. Diethylcarbamazine. E. Niclosamide.

Correct answer = C. The symptoms and other findings for this patient are consistent with neurocysticercosis. Albendazole is the drug of choice for the treatment of this infestation. The other drugs are not effective against the larval forms of tapeworms.

All of the following drugs may be beneficial in the treatment of patients with acromegaly except: A. Lanreotide. B. Octreotide. C. Pegvisomant. D. Somatropin.

Correct answer = D. Acromegaly is characterized by an excess of GH. Somatropin is synthetic human GH, so it would not be beneficial in this setting. Lanreotide and octreotide are synthetic analogs of somatostatin, which inhibits GH. Pegvisomant is an antagonist at GH receptors and is used to treat acromegaly.

All of the following are strategies to minimize the development of HPA axis suppression with corticosteroid therapy except: A. Alternate-day administration of therapy. B. Administration via topical or inhalation routes when possible. C. Using the lowest dose of corticosteroid that adequately controls symptoms. D. Administration of two-thirds of the daily dose in the morning and one-third in the afternoon.

Correct answer = D. Administration of two-thirds of the dose in the morning and one-third in the afternoon is a strategy to mimic the normal diurnal variation of cortisol secretion. However, it is not a strategy to prevent suppression of the HPA axis. All of the other methods will help prevent the likelihood of suppression of the HPA axis.

Which of the following antibiotics is considered safe to use in neonates? A. Chloramphenicol. B. Sulfamethoxazole/trimethoprim. C. Tetracycline. D. Penicillin G.

Correct answer = D. Chloramphenicol and sulfonamides (sulfamethoxazole) can cause toxic effects in newborns due to poorly developed renal and hepatic elimination processes. Tetracycline can have effects on bone growth and development and should be avoided in newborns and young children. Penicillin G is safe and effective in this population.

Which of the following antibiotics exhibits concentrationdependent killing? A. Clindamycin. B. Linezolid. C. Vancomycin. D. Daptomycin.

Correct answer = D. Clindamycin, linezolid, and vancomycin exhibit time-dependent killing, while daptomycin works best in a concentration-dependent fashion.

A female patient who is being treated for chronic hepatitis B develops nephrotoxicity while on treatment. Which is the most likely medication she is taking for HBV treatment? A. Entecavir. B. Telbivudine. C. Lamivudine. D. Adefovir.

Correct answer = D. Nephrotoxicity is the most commonly seen with adefovir.

A 24-year-old male has returned to the clinic for his 1-month check-up after starting treatment for tuberculosis. He is receiving isoniazid, rifampin, pyrazinamide, and ethambutol. He states he feels fine, but now is having difficulty reading his morning newspaper and feels he may need to get glasses. Which of the following drugs may be causing his decline in vision? A. Isoniazid. B. Rifampin. C. Pyrazinamide. D. Ethambutol.

Correct answer = D. Optic neuritis, exhibited as a decrease in visual acuity or loss of color discrimination, is the most important side effect associated with ethambutol. Visual disturbances generally are dose related and more common in patients with reduced renal function. They are reversible (weeks to months) if ethambutol is discontinued promptly

An 18-year-old female presents to the urgent care clinic with urinary frequency, urgency, and fever for the past 3 days. Based on symptoms and a urinalysis, she is diagnosed with a urinary tract infection. Cultures reveal Enterococcus faecalis that is pan sensitive. Which of the following is an appropriate oral option to treat the urinary tract infection in this patient? A. Cephalexin. B. Vancomycin. C. Cefdinir. D. Amoxicillin.

Correct answer = D. Option A and C are incorrect because enterococci are inherently resistant to all cephalosporins. Option B is incorrect because oral vancomycin is not absorbed and would not reach the urinary tract in sufficient quantities to treat a urinary tract infection. Option D is the best choice, as amoxicillin is well absorbed orally and concentrates in the urine

Which of the following adverse effects is associated with daptomycin? A. Ototoxicity. B. Red man syndrome. C. QTc prolongation. D. Rhabdomyolysis.

Correct answer = D. Ototoxicity and red man syndrome are associated with vancomycin. QTc prolongation is associated with telavancin. Myalgias and rhabdomyolysis have been reported with daptomycin therapy and require patient education and monitoring.

A 56-year-old man from South America is found to be parasitized by both schistosomes and T. solium—the pork tapeworm. Which of the following anthelmintic drugs would be effective for both infestations? A. Albendazole. B. Ivermectin. C. Mebendazole. D. Praziquantel.

Correct answer = D. Praziquantel is a primary drug for the treatment of trematode and cestode infestations. Although albendazole is effective in cysticercosis, it is not active against flukes, and this patient has no evidence of cysticercosis. Ivermectin and mebendazole treat nematode infestations.

A 35-year-old male, formerly a heroin abuser, has been on methadone maintenance for the last 13 months. Two weeks ago, he had a positive tuberculosis skin test (PPD test), and a chest radiograph showed evidence of right upper lobe infection. He was started on standard four-drug antimycobacterial therapy. He has come to the emergency department complaining of "withdrawal symptoms." Which of the following antimycobacterial drugs is likely to have caused this patient's acute withdrawal reaction? A. Ethambutol. B. Isoniazid. C. Pyrazinamide. D. Rifampin. E. Streptomycin.

Correct answer = D. Rifampin is a potent inducer of cytochrome P450-dependent drug-metabolizing enzymes. The duration of action of methadone is dependent upon hepatic clearance, so enhanced drug metabolism will shorten the duration and increase the risk of withdrawal symptoms in individuals on methadone maintenance. None of the other drugs listed induce cytochrome P450 enzymes.

A 64-year-old woman with a history of type 2 diabetes is diagnosed with heart failure. Which of the following medications would be a poor choice for controlling her diabetes? A. Exenatide. B. Glyburide. C. Nateglinide. D. Pioglitazone. E. Sitagliptin

Correct answer = D. The TZDs (pioglitazone and rosiglitazone) can cause fluid retention and lead to a worsening of heart failure. They should be used with caution and dose reduction, if at all, in patients with heart failure. Exenatide, glyburide, nateglinide, and sitagliptin do not have precautions for use in heart failure patients.

A 72-year-old male presents with fever, cough, malaise, and shortness of breath. His chest x-ray shows bilateral infiltrates consistent with pneumonia. Bronchial wash cultures reveal Pseudomonas aeruginosa sensitive to cefepime. Which of the following is the best dosing scheme for cefepime based on the drug's timedependent bactericidal activity? A. 1 g every 6 hours given over 30 minutes. B. 2 g every 12 hours given over 3 hours. C. 4 g every 24 hours given over 30 minutes. D. 4 g given as continuous infusion over 24 hours.

Correct answer = D. The clinical efficacy of cefepime is based on the percentage of time that the drug concentration remains above the MIC. A continuous infusion would allow for the greatest amount of time above the MIC compared to intermittent (30 minutes) and prolonged infusions (3 to 4 hours).

Which of the following diabetes medications is most appropriately paired with an adverse effect associated with its use? A. Canagliflozin—lactic acidosis. B. Metformin—urinary tract infections. C. Nateglinide—heart failure. D. Liraglutide—pancreatitis.

Correct answer = D. The incretin mimetics are associated with a risk of pancreatitis. Lactic acidosis is a rare but serious side effect of metformin (not canagliflozin). Adverse effects of canagliflozin are genital mycotic infections, urinary tract infections, and urinary frequency. Nateglinide may cause hypoglycemia but has not been associated with heart failure. The TZDs have been associated with heart failure.

A 24-year-old pregnant female presents to the urgent care clinic with fever, frequency, and urgency. She is diagnosed with a urinary tract infection (UTI). Based on potential harm to the fetus, which of the following medications should be avoided in treating her UTI? A. Nitrofurantoin. B. Amoxicillin. C. Cephalexin. D. Tobramycin.

Correct answer = D. Tobramycin (an aminoglycoside) is considered a pregnancy risk category D drug which means there is chance for potential harm to the fetus. Nitrofurantoin, amoxicillin (a penicillin), and cephalexin (a cephalosporin) are considered category B.

A fellow health care provider is concerned about prescribing orlistat to his adolescent patients. He understands that it is approved for adolescents aged 12 years and older and, therefore, prescribes this medication more often than any other for adolescent obesity. Unfortunately, many of his adolescent patients are stopping the medication during the first month of treatment. Which of the following side effects is the most likely reason these adolescents are stopping the drug? A. Tachycardia. B. Valvulopathy. C. Suicidal ideation. D. Drowsiness. E. Flatulence.

Correct answer = E. Flatulence is a very common side effect with orlistat, along with several other GI disturbances. For adolescents, these side effects may be embarrassing and difficult to manage. It is important to counsel patients about these gastrointestinal side effects with orlistat and recommend a low-fat diet as well as cholestyramine to counteract the effects should they become bothersome. The other side effects listed have been seen with other obesity medications, but not with orlistat.

All of the following agents are correctly paired with an appropriate clinical use of the drug except: A. Desmopressin—treatment of diabetes insipidus. B. hCG—treatment of infertility. C. Octreotide—treatment of bleeding esophageal varices. D. Oxytocin—induction of labor. E. Goserelin—growth hormone deficiency.

Correct answer = E. Goserelin is a GnRH analog that may be used for the treatment of prostate cancer or endometriosis. The other choices are paired correctly with their respective clinical uses.

A 38-year-old obese male with depression is considering a weight loss medication following several failed attempts with diet and exercise. Which of the following medications should be avoided in this individual? A. Phentermine. B. Phentermine/topiramate. C. Orlistat. D. Diethylpropion. E. Lorcaserin.

Correct answer = E. Lorcaserin may cause suicidal ideation and would not be advisable for an individual with depression. Also, he is likely on a medication that may increase serotonin levels. The addition of lorcaserin, a serotonin receptor agonist, could lead to serotonin syndrome. Therefore, avoidance of the combination is advisable.

Diethylstilbestrol (DES) should never be used in pregnant women because it is associated with which of the following? (A) Deep vein thrombosis (B) Feminization of the external genitalia of male offspring (C) Infertility and development of vaginal cancer in female offspring (D) Miscarriages (E) Virilization of the external genitalia of female offspring

Diethylstilbestrol (DES) is a nonsteroidal estrogen agonist. Several decades ago, misguided use of the drug in pregnant women appears to have resulted in fetal damage that predisposed female offspring to infertility and a rare form of vaginal cancer. For this reason, the drug should be avoided in pregnant women. Other estrogenic drugs do not appear to have these effects. Although estrogens do increase the risk of deep vein thrombosis, this is not the reason why DES should be avoided. The answer is C.

In a patient suffering from pseudomembranous colitis due to C difficile with established hypersensitivity to metronidazole the most likely drug to be of clinical value is (A) Amoxicillin (B) Chloramphenicol (C) Doxycycline (D) Levofloxacin (E) Vancomycin

Disturbances of gut flora occur commonly during treatment with antibiotics, and pseudomembranous colitis has been associated with the use of many agents including ampicillin and clindamycin. Vancomycin can be used in treatment of pseudomembranous colitis in patients with established hypersensitivity to metronidazole. The answer is E.

Which adverse effect occurs with the use of albendazole during intestinal nematode therapy? (A) Cholestatic jaundice (B) Corneal opacities (C) Hirsutism (D) Peripheral neuropathy (E) None of the above

Doses of albendazole required for intestinal nematode therapy are almost free of adverse effects even in the malnourished or debilitated patient. Gastrointestinal distress may occur in children with ascariasis who are heavily parasitized, together with a slight headache or dizziness. Avoid the drug in children younger than 2 yr because of rare reports of seizures. The answer is E.

A 36-year-old woman recently treated for leukemia is admitted to the hospital with malaise, chills, and high fever. Gram stain of blood reveals the presence of gram-negative bacilli. The initial diagnosis is bacteremia, and parenteral antibiotics are indicated. The records of the patient reveal that she had a severe urticarial rash, hypotension, and respiratory difficulty after oral penicillin V about 6 mo ago. The most appropriate drug regimen for empiric treatment is (A) Aztreonam (B) Ceftriaxone (C) Meropenem (D) Oxacillin (E) Ticarcillin plus clavulanic acid

Each of the drugs listed has activity against some gram-negative bacilli. All penicillins should be avoided in patients with a history of allergic reactions to any individual penicillin drug. Cephalosporins should also be avoided in patients who have had anaphylaxis or other severe hypersensitivity reactions after use of a penicillin. There is partial cross-reactivity between penicillins and the carbapenems such as imipenem and meropenem, but no crossreactivity between the penicillins and aztreonam. The answer is A.

All the following agents have been used in drug regimens for the treatment of breast carcinoma. Which one has specific activity in a subset of female breast cancers? (A) Cyclophosphamide (B) Doxorubicin (C) Fluoxymesterone (D) Methotrexate (E) Trastuzumab

Each of the drugs listed has been used in drug regimens for breast cancer, but only trastuzumab has specificity in its actions. The drug is a monoclonal antibody to a surface protein in breast cancer cells that overexpress the HER-2 protein. Consequently, trastuzumab has value in a specific subset of breast cancers. The answer is E.

A 23-year-old woman desires a combined oral contraceptive for pregnancy protection. Which of the following patient factors would lead a health professional to recommend an alternative form of contraception? (A) Evidence of hirsutism (B) History of gastroesophageal reflux disease and is currently taking omeprazole (C) History of pelvic inflammatory disease (D) History of migraine headache that is well controlled by sumatriptan (E) She plans to use this contraceptive for about 1 yr and will then attempt to become pregnant

Estrogen-containing hormonal contraceptives increase the risk of episodes of migraine headache. The answer is D.

Finasteride has efficacy in the prevention of male-pattern baldness by virtue of its ability to do which of the following? (A) Competitively antagonize androgen receptors (B) Decrease the release of gonadotropins (C) Increase the serum concentration of sex hormone-binding globulin (D) Inhibit the synthesis of testosterone (E) Reduce the production of dihydrotestosterone

Finasteride inhibits 5α-reductase, the enzyme that converts testosterone to DHT, the principal androgen in androgensensitive hair follicles. The answer is E.

Which of the following statements about beta-lactam antibiotics is false? (A) Cephalexin and other first-generation cephalosporins do not cross the blood-brain barrier (B) Ceftriaxone and nafcillin are both eliminated mainly via biliary secretion (C) Instability of penicillins in gastric acid can limit their oral absorption (D) Renal tubular reabsorption of amoxicillin is inhibited by probenecid (E) Ticarcillin has activity against several gram negative rods

First- and second-generation cephalosporins are not effective in meningitis because they do not readily enter the cerebrospinal fluid. The elimination half-lives of many beta-lactam antibiotics are prolonged by probenecid, which inhibits their proximal tubular secretion. The answer is D.

Regarding the toxicity of aminoglycosides which statement is accurate? (A) Gentamicin and tobramycin are the least likely to cause renal damage (B) Ototoxicity due to amikacin and gentamicin includes vestibular dysfunction, which is often irreversible (C) Ototoxicity is reduced if loop diuretics are used to facilitate the renal excretion of aminoglycoside antibiotics (D) Reduced blood creatinine is an early sign of aminoglycoside nephrotoxicity (E) Skin reactions are very rare following topical use of neomycin

Gentamicin and tobramycin are the most nephrotoxic aminoglycosides. The incidence of nephrotoxic effects with gentamicin is 2 to 3 times greater than the incidence of ototoxicity. With traditional dosage regimens, the first indication of potential nephrotoxicity is an increase in trough serum levels of aminoglycosides, which is followed by an increase in blood creatinine. Although aminoglycoside ototoxicity usually involves irreversible effects on vestibular function, hearing loss can also occur. Ototoxicity is enhanced by loop diuretics. Skin reactions are common with topical use of neomycin. The answer is B.

After a backpacking trip in the mountains, a 24-year-old man develops diarrhea. He acknowledges drinking stream water without purification, and you suspect he is showing symptoms of giardiasis. Because you know that laboratory detection of cysts or trophozoites in the feces can be difficult, you decide to treat the patient empirically with (A) Chloroquine (B) Emetine (C) Pentamidine (D) Tinidazole (E) TMP-SMZ

Giardiasis is a common intestinal protozoan infection caused by Giardia lamblia. A large number of infections result from fecal contamination of food or water. Metronidazole is frequently used, but tinidazole is equally effective. The answer is D.

Which one of the following drugs promotes the release of endogenous insulin? (A) Acarbose (B) Canagliflozin (C) Glipizide (D) Metformin (E) Miglitol (F) Pioglitazone

Glipizide is a second-generation sulfonylurea that promotes insulin release by closing potassium channels in pancreatic B cells. The answer is C.

Which of the following patients is most likely to be treated with intravenous glucagon? (A) An 18-year-old woman who took an overdose of cocaine and now has a blood pressure of 190/110 mm Hg (B) A 27-year-old woman with severe diarrhea caused by a flare in her inflammatory bowel disease (C) A 57-year-old woman with type 2 diabetes who has not taken her glyburide for the last 3 d (D) A 62-year-old man with severe bradycardia and hypotension resulting from ingestion of an overdose of atenolol (E) A 74-year-old man with lactic acidosis as a complication of severe infection and shock

Glucagon acts through cardiac glucagon receptors to stimulate the rate and force of contraction of the heart. Because this bypasses cardiac β adrenoceptors, glucagon is useful in the treatment of β-blocker-induced cardiac depression. The answer is D.

If ampicillin and piperacillin are used in combination in the treatment of infections resulting from Pseudomonas aeruginosa, antagonism may occur. The most likely explanation is that (A) Ampicillin is bacteriostatic (B) Ampicillin induces beta-lactamase production (C) Autolytic enzymes are inhibited by piperacillin (D) Piperacillin blocks the attachment of ampicillin to penicillin-binding proteins (E) The 2 drugs form an insoluble complex

Gram-negative rods such as Enterobacter and Pseudomonas aeruginosa have inducible beta-lactamases. Several betalactam antibiotics, including ampicillin, cefoxitin, and imipenem, are potent inducers of beta-lactamase production. When such inducers are used with a hydrolyzable penicillin (eg, piperacillin), antagonism may result. The answer is B

Which drug is least likely to be effective in the treatment of esophageal candidiasis if it is used by the oral route? (A) Clotrimazole (B) Griseofulvin (C) Ketoconazole (D) Itraconazole (E) Nystatin

Griseofulvin has no activity against C albicans and is not effective in the treatment of systemic or superficial infections caused by such organisms. "Swish and swallow" formulations of clotrimazole and nystatin have been used commonly. Most of the azoles are effective in esophageal candidiasis. The answer is B.

Which of the following is an important effect of insulin? (A) Increased conversion of amino acids into glucose (B) Increased gluconeogenesis (C) Increased glucose transport into cells (D) Inhibition of lipoprotein lipase (E) Stimulation of glycogenolysis

Insulin lowers serum glucose concentration in part by driving glucose into cells, particularly into muscle cells. The answer is C.

A patient with AIDS has an extremely high viral RNA titer. While blood is being drawn from this patient, the syringe is accidentally dropped, contaminating the floor, which is made of porous material. The best way to deal with this is to (A) Clean the floor with a 10% solution of household bleach (B) Clean the floor with soap and water (C) Completely replace the contaminated part of the floor (D) Neutralize the spill with a solution of potassium permanganate (E) Seal the room and decontaminate with ethylene oxide

Household bleach contains sodium hypochlorite. A 1:10 dilution of bleach is effective for disinfection of a direct blood spill on a porous surface. In addition to inactivating HIV, sodium hypochlorite solutions have disinfectant activity against other viruses, including hepatitis B virus. The answer is A.

10. Which statement about "once-daily" dosing with aminoglycosides is not accurate? (A) Dose adjustment is less important in renal dysfunction (B) It is convenient for outpatient treatment (C) Less nursing time is required for drug administration (D) Often less side effects than multiple (conventional) dosing regimens (E) Underdosing is less of a problem

In "once-daily dosing" with aminoglycosides, the selection of an appropriate dose is particularly critical in patients with renal insufficiency. The aminoglycosides are eliminated by the kidney in proportion to creatinine clearance. Knowledge of the degree of insufficiency, based on plasma creatinine (or BUN), is essential for estimation of the appropriate single daily dose of an aminoglycoside. The answer is A.

A young woman complains of abdominal pain at the time of menstruation. Careful evaluation indicates the presence of significant endometrial deposits on the pelvic peritoneum. Which of the following is the most appropriate medical therapy for this patient? (A) Flutamide, orally (B) Medroxyprogesterone acetate by intramuscular injection (C) Norgestrel as an IUD (D) Oxandrolone by intramuscular injection (E) Raloxifene orally

In endometriosis, suppression of ovarian function and production of gonadal steroids are useful. Intramuscular injection of relatively large doses of medroxyprogesterone provides 3 months of an ovarian suppressive effect because of inhibition of pituitary production of gonadotropins. The answer is B.

Which of the following is a sign or symptom that would be expected to occur in the event of chronic overdose with exogenous T4? (A) Bradycardia (B) Dry, puffy skin (C) Large tongue and drooping of the eyelids (D) Lethargy, sleepiness (E) Weight loss

In hyperthyroidism, the metabolic rate increases, and even though there is increased appetite, weight loss often occurs. The other choices are symptoms seen in hypothyroidism. The answer is E.

Men who use large doses of anabolic steroids are at increased risk of which of the following? (A) Anemia (B) Cholestatic jaundice and elevation of aspartate transaminase levels in the blood (C) Hirsutism (D) Hyperprolactinemia (E) Testicular enlargement

In men, large doses of anabolic steroids are associated with liver impairment, including cholestasis and elevation of serum concentrations of transaminases. The answer is B.

A patient needs antibiotic treatment for native valve, culturepositive infective enterococcal endocarditis. His medical history includes a severe anaphylactic reaction to penicillin G during the last year. The best approach would be treatment with (A) Amoxicillin-clavulanate (B) Aztreonam (C) Ceftriaxone (D) Ticarcillin (E) Vancomycin

In patients who have had a severe reaction to a penicillin, it is inadvisable to administer a cephalosporin or a carbapenem such as meropenem. Aztreonam has no significant activity against gram-positive cocci, so the logical treatment in this case is vancomycin, often with an aminoglycoside (eg, gentamicin) for synergistic activity against enterococci. The answer is E.

A 3-year-old girl presented with hirsutism, breast enlargement, and a height and bone age that was consistent with an age of 9. Diagnostic testing revealed precocious puberty. Which of the following is the most appropriate drug for treatment of this patient's precocious puberty? (A) Atosiban (B) Follitropin (C) Leuprolide (D) Octreotide (E) Pegvisomant

In precocious puberty, the hypothalamic-pituitary-gonadal axis becomes prematurely active for reasons that are not understood. Treatment involves suppressing gonadotropin secretion with continuous administration of a long-acting GnRH agonist such as leuprolide. The answer is C.

Which is the drug of choice if this patient is suffering from persistent lung lesions or disseminated disease caused by Coccidioides immitis? (A) Amphotericin B (B) Flucytosine (C) Itraconazole (D) Micofungin (E) Terbinafine

In progressive or disseminated forms of coccidioidomycosis, systemic antifungal drug treatment is needed. Until recently, amphotericin B was the recommended therapy, but fluconazole or itraconazole are now generally preferred. Note that the risk of dissemination is much greater in African Americans (10% incidence) and in pregnant women during the third trimester. The answer is C.

This drug can clear trypanosomes from the blood and lymph nodes and is active in the late CNS stages of African sleeping sickness. (A) Emetine (B) Melarsoprol (C) Nifurtimox (D) Pentamidine (E) Suramin

In the advanced stages of African sleeping sickness, melarsoprol is the drug of choice because, unlike pentamidine or suramin, it effectively enters the CNS. Nifurtimox is the most commonly used drug for Chagas' disease. The answer is B.

A 56-year-old woman presented to the emergency department with tachycardia, shortness of breath, and chest pain. She had had shortness of breath and diarrhea for the last 2 d and was sweating and anxious. A relative reported that the patient had run out of methimazole 2 wk earlier. A TSH measurement revealed a value of <0.01 mIU/L (normal 0.4-4.0 mIU/L). The diagnosis of thyroid storm was made. Which of the following is a drug that is a useful adjuvant in the treatment of thyroid storm? (A) Amiodarone (B) Betamethasone (C) Epinephrine (D) Propranolol (E) Radioactive iodine

In thyroid storm, β blockers such as propranolol are useful in controlling the tachycardia and other cardiac abnormalities, and propranolol also inhibits peripheral conversion of T4 to T3. The answer is D.

A 24-year-old woman with type 1 diabetes wishes to try tight control of her diabetes to improve her long-term prognosis. Which of the following regimens is most appropriate? (A) Morning injections of mixed insulin lispro and insulin aspart (B) Evening injections of mixed regular insulin and insulin glargine (C) Morning and evening injections of regular insulin, supplemented by small amounts of NPH insulin at mealtimes (D) Morning injections of insulin glargine, supplemented by small amounts of insulin lispro at mealtimes (E) Morning injection of NPH insulin and evening injection of regular insulin

Insulin regimens for tight control usually take the form of establishing a basal level of insulin with a small amount of a longacting preparation (eg, insulin glargine) and supplementing the insulin levels, when called for by food intake, with short-acting insulin lispro. Less tight control may be achieved with 2 injections of intermediate-acting insulin per day. Because intake of glucose is mainly during the day, long-acting insulins are usually given in the morning, not at night. The answer is D.

A 65-year-old man with multinodular goiter is scheduled for a near-total thyroidectomy. Which of the following drugs will be administered for 10-14 d before surgery to reduce the vascularity of his thyroid gland? (A) Levothyroxine (B) Liothyronine (C) Lugol's solution (D) Prednisone (E) Radioactive iodine

Iodides inhibit the synthesis and release of thyroid hormone and decrease the size and vascularity of the hyperplastic gland. Lugol's solution contains a mixture of potassium iodide and iodine. The answer is C.

A missionary from the United States is sent to work in a geographic region of a Central American country where Onchocerca volvulus is endemic. Infections resulting from this tissue nematode (onchocerciasis) are a cause of "river blindness," because microfilariae migrate through subcutaneous tissues and concentrate in the eyes. Which drug should be used prophylactically to prevent onchocerciasis? (A) Albendazole (B) Diethylcarbamazine (C) Ivermectin (D) Oxamniquine (E) Pyrantel pamoate

Ivermectin prevents onchocerciasis and is the drug of choice in the individual and mass treatment of the disease. The only other drug listed with any activity against Onchocerca volvulus is diethylcarbamazine. Suramin (not listed) also has activity against onchocerciasis, but like diethylcarbamazine, it is less effective and more toxic than ivermectin. The answer is C.

Serious cardiac effects have occurred when this drug was taken by patients using the antihistamines astemizole or terfenadine (A) Amphotericin B (B) Griseofulvin (C) Ketoconazole (D) Terbinafine (E) Voriconazole

Ketoconazole was the first oral azole introduced into clinical use, but it has a greater propensity to inhibit human cytochrome P450 enzymes than other azoles and is no longer widely used in the United States. Cardiotoxicity may occur when ketoconazole is used by patients taking astemizole or terfenadine as a result of the ability of ketoconazole to inhibit their metabolism via hepatic cytochromes P450. The answer is C.

Which of the following statements about interferon-α is false? (A) At the start of treatment, most patients experience flulike symptoms (B) Indications include treatment of genital warts (C) It is used in the management of hepatitis B and C (D) Lamivudine interferes with its activity against hepatitis B (E) Toxicity includes bone marrow suppression

Lamivudine is used in monotherapy of HBV infections and does not oppose the beneficial effects of interferon-α when both agents are used together in the treatment of hepatitis B. The answer is D.

A 60-year-old man is found to have a prostate lump and an elevated prostate-specific antigen (PSA) blood test. Magnetic resonance imaging suggests several enlarged lymph nodes in the lower abdomen, and an x-ray reveals 2 radiolucent lesions in the bony pelvis. This patient is likely to be treated with which of the following drugs? (A) Anastrozole (B) Desogestrel (C) Leuprolide (D) Methyltestosterone (E) Oxandrolone

Leuprolide is a GnRH agonist used in the treatment of men with prostate cancer. Continuous use leads to downregulation of testosterone production. Initially, the agonist action increases testosterone, causing a tumor flare. To prevent this, flutamide, a competitive antagonist of the androgen receptor, is added until downregulation of testosterone is complete. The answer is C.

An important difference between leuprolide and ganirelix is that ganirelix (A) Can be administered as an oral formulation (B) Can be used alone to restore fertility to hypogonadal men and women (C) Immediately reduces gonadotropin secretion (D) Initially stimulates pituitary production of LH and FSH (E) Must be administered in a pulsatile fashion

Leuprolide is an agonist of GnRH receptors, whereas ganirelix is an antagonist. Although both drugs can be used to inhibit gonadotropin release, ganirelix does so immediately, whereas leuprolide does so only after about 1 wk of sustained activity. The answer is C.

Regarding the clinical use of liposomal formulations of amphotericin B, which statement is accurate? (A) Amphotericin B affinity for these lipids is greater than affinity for ergosterol (B) Less expensive to use than conventional amphotericin B (C) More effective in fungal infections because they increase tissue uptake of amphotericin B (D) They decrease the nephrotoxicity of amphotericin B (E) They have a wider spectrum of antifungal activity than conventional formulations of amphotericin B

Liposomal formulations of amphotericin B result in decreased accumulation of the drug in tissues, including the kidney. As a result, nephrotoxicity is decreased. With some lipid formulations, infusion-related toxicity may also be reduced. Lipid formulations do not have a wider antifungal spectrum; their daily cost ranges from 10 to 40 times more than the conventional formulation of amphotericin B. The answer is D.

A malnourished 12-year-old child who lives in a rural area of the southern United States presents with weakness, fever, cough, abdominal pain, and eosinophilia. His mother tells you that she has seen long, thin worms in the child's stools, sometimes with blood. A presumptive diagnosis of ascariasis is confirmed by the presence of the ova of A lumbricoides in the stools. However, microscopy also reveals that the stools contain the eggs of Necator americanus. The drug most likely to be effective in the treatment of this child is (A) Diethylcarbamazine (B) Ivermectin (C) Mebendazole (D) Niclosamide (E) Praziquantel

Mebendazole is effective against both nematodes causing infection in this child. Albendazole and pyrantel pamoate (not listed in this question) are also primary drugs for the treatment of combined infections due to hookworm and roundworm. The answer is C.

The primary mechanism of resistance of gram-positive organisms to macrolide antibiotics including erythromycin is (A) Changes in the 30S ribosomal subunit (B) Decreased drug permeability of the cytoplasmic membrane (C) Formation of drug-inactivating acetyltransferases (D) Formation of esterases that hydrolyze the lactone ring (E) Methylation of binding sites on the 50S ribosomal subunit

Methylase production and methylation of the receptor site are established mechanisms of resistance of gram-positive organisms to macrolide antibiotics. Such enzymes may be inducible by macrolides or constitutive; in the latter case, cross-resistance occurs between macrolides and clindamycin. Increased expression of efflux pumps is also a mechanism of macrolide resistance. Esterase formation is a mechanism of macrolide resistance seen in coliforms. The answer is E.

10. Metronidazole is not effective in the treatment of (A) Amebiasis (B) Infections due to Bacteroides fragilis (C) Infections due to Pneumocystis jiroveci (D) Pseudomembranous colitis (E) Trichomoniasis

Metronidazole is the drug of first choice for all of the conditions listed except pneumocystosis. The answer is C.

A teenager seeks postcoital contraception. Which of the following preparations will be effective for this purpose? (A) Clomiphene (B) Ethinyl estradiol (C) Diethylstilbestrol (DES) (D) Mifepristone (E) Norgestrel

Mifepristone, an antagonist at progesterone and glucocorticoid receptors, has a luteolytic effect and is effective as a postcoital contraceptive. When combined with a prostaglandin, it is also an effective abortifacient. The answer is D.

A 72-kg patient with creatinine clearance of 80 mL/min has a gram-negative infection. Amikacin is administered intramuscularly at a dose of 5 mg/kg every 8 h, and the patient begins to respond. After 2 d, creatinine clearance declines to 40 mL/min. Assuming that no information is available about amikacin plasma levels, what would be the most reasonable approach to management of the patient at this point? (A) Administer 5 mg/kg every 12 h (B) Decrease the dosage to daily total of 200 mg (C) Decrease the dosage to 180 mg every 8 h (D) Discontinue amikacin and switch to gentamicin (E) Maintain the patient on the present dosage and test auditory function

Monitoring plasma drug levels is important when aminoglycosides are used. In this case, the patient seems to be improving, so a decrease of the amikacin dose in proportion to decreased creatinine clearance is most appropriate. Because creatinine clearance is only one half of the starting value, a dose reduction should be made to one half of that given initially. The answer is C.

Risk factors for multidrug-resistant tuberculosis include (A) A history of treatment of tuberculosis without rifampin (B) Recent immigration from Asia and living in an area of over 4% isoniazid resistance (C) Recent immigration from Latin America (D) Residence in regions where isoniazid resistance is known to exceed 4% (E) All of the above

Multidrug-resistant tuberculosis (MDR-TB) is defined as resistance to 2 or more drugs. All the risk factors are relevent. In the case of resistance to both INH and rifampin, initial regimens still include both drugs, plus ethambutol, pyrazinamide, streptomycin (or other aminoglycoside), and a fluoroquinolone. Continuation therapy should include at least 3 drugs shown to be active in vitro against the infecting strain. The appropriate duration of therapy has not been established. The answer is E.

An adult patient is being treated for acute leukemia with a combination of anticancer drugs that includes cyclophosphamide, mercaptopurine, methotrexate, vincristine, and prednisone. He is also using ondansetron for emesis, a chlorhexidine mouthwash to reduce mucositis, and laxatives. The patient complains of "pins and needle" sensations in the extremities and muscle weakness. He is not able to execute a deep knee bend or get up out of a chair without using his arm muscles. He is also very constipated. If these problems are related to the chemotherapy, which of the following is the most likely causative agent? (A) Cyclophosphamide (B) Mercaptopurine (C) Methotrexate (D) Prednisone (E) Vincristine

Neuropathy is a toxic side effect of vincristine. In its mildest form, paresthesias occur, but it progresses to significant muscle weakness, initially in the quadriceps muscle group. Constipation is the most common symptom of autonomic neuropathy. The answer is E.

Which antiseptic promotes wound healing? (A) Cetylpyridinium chloride (B) Chlorhexidine (C) Hexachlorophene (D) Phenol (E) None of the above

No antiseptic in current use is able to promote wound healing, and most agents do the opposite. In general, cleansing of abrasions and superficial wounds with soap and water is just as effective as and less damaging than the application of topical antiseptics. Phenol is only used as a disinfectant of inanimate objects! The answer is E.

A 47-year-old man exhibited signs and symptoms of acromegaly. Radiologic studies indicated the presence of a large pituitary tumor. Surgical treatment of the tumor was only partially effective in controlling his disease. At this point, which of the following drugs is most likely to be used as pharmacologic therapy? (A) Cosyntropin (B) Desmopressin (C) Leuprolide (D) Octreotide (E) Somatropin

Octreotide, a somatostatin analog, has some efficacy in reducing the excess GH production that causes acromegaly. The answer is D.

Which compound is the safest drug to use topically to treat scabies and pediculosis? (A) Benzoyl peroxide (B) Chlorhexidine (C) Lindane (D) Permethrin (E) Silver sulfadiazine

Of the agents listed, both lindane and permethrin are effective scabicides and pediculicides. However, there is some concern about systemic absorption of lindane, which may cause neurotoxicity and hematotoxicity. Accidental ingestion of lindane in children has caused seizures. The answer is D.

A 37-year-old woman with infertility due to obstructed fallopian tubes was undergoing ovulation induction in preparation for in vitro fertilization. After 10 d of treatment with leuprolide, the next step in the procedure is most likely to involve 10-14 d of treatment with which of the following? (A) Bromocriptine (B) Follitropin (C) Gonadorelin (D) hCG (E) Pergolide

Once the patient's endogenous gonadotropin production has been inhibited through continuous administration of the GnRH agonist leuprolide, the next step in ovulation induction is the administration of a drug with FSH activity to stimulate follicle maturation. Follitropin is recombinant FSH. The only other drug listed that is used in ovulation induction is hCG, but this is an LH analog. The answer is B.

A 29-year-old woman in her 41st wk of gestation had been in labor for 12 h. Although her uterine contractions had been strong and regular initially, they had diminished in force during the past hour. Which of the following agents would be used to facilitate this woman's labor and delivery? (A) Dopamine (B) Leuprolide (C) Oxytocin (D) Prolactin (E) Vasopressin

Oxytocin is an effective stimulant of uterine contraction that is routinely used to augment labor. The answer is C.

When initiating T4 therapy for an elderly patient with longstanding hypothyroidism, it is important to begin with small doses to avoid which of the following? (A) A flare-up of exophthalmos (B) Acute renal failure (C) Hemolysis (D) Overstimulation of the heart (E) Seizures

Patients with longstanding hypothyroidism, especially those who are elderly, are highly sensitive to the stimulatory effects of T4 on cardiac function. Administration of regular doses can cause overstimulation of the heart and cardiac collapse. The answer is D.

The primary mechanism of antibacterial action of the penicillins involves inhibition of (A) Beta-lactamases (B) Cell membrane synthesis (C) N-acetylmuramic acid synthesis (D) Peptidoglycan cross-linking (E) Transglycosylation

Penicillins (and cephalosporins) bind to PBPs acting at the transpeptidation stage of cell wall synthesis (the final step) to inhibit peptidoglycan cross-linking. The beta-lactam antibiotics also activate autolysins, which break down the bacterial cell wall. Synthesis of N-acetylmuramic acid is inhibited by fosfomycin. Vancomycin inhibits transglycolase, preventing elongation of peptidoglycan chains. The answer is D.

A 7-year-old boy underwent successful chemotherapy and cranial radiation for treatment of acute lymphocytic leukemia. One month after the completion of therapy, the patient presented with excessive thirst and urination plus hypernatremia. Laboratory testing revealed pituitary diabetes insipidus. To correct these problems, this patient is likely to be treated with which of the following? (A) Corticotropin (B) Desmopressin (C) hCG (D) Menotropins (E) Thyrotropin

Pituitary diabetes insipidus results from deficiency in vasopressin. It is treated with desmopressin, a peptide agonist of vasopressin V2 receptors. The answer is B.

Which statement about antiprotozoal drugs is accurate? (A) Chloroquine is an inhibitor of plasmodial dihydrofolate reductase (B) Mefloquine destroys secondary exoerythrocytic schizonts (C) Primaquine is a blood schizonticide and does not affect secondary tissue schizonts (D) Proguanil complexes with double-stranded DNA-blocking replication (E) Trimethoprim-sulfamethoxazole is the drug of choice for Pneumocystis jiroveci pneumonia

Proguanil (not chloroquine) is an inhibitor of dihydrofolate reductase. Primaquine (not mefloquine) is the drug that destroys secondary exoerythrocytic schizonts. TMP-SMZ is the drug of choice for Pneumocystis jiroveci pneumonia. The answer is E.

A 25-year-old woman presents with insomnia and fears she may have "something wrong with her heart." She describes "her heart jumping out of her chest." She feels healthy otherwise and reports she has lots of energy. Lab tests confirm hyperthyroidism. Which of the following is a drug that produces a permanent reduction in thyroid activity? (A) 131I (B) Methimazole (C) Propylthiouracil (D) Thiocyanate (SCN-) (E) Thyroglobulin

Propylthiouracil and, to a much lesser extent, methimazole inhibit peripheral conversion of T4 to T3. Thyroglobulin is not a drug. Radioactive iodine is the only medical therapy that produces a permanent reduction of thyroid activity. Anions such as thiocyanate (SCN-) and perchlorate (ClO4-) block the uptake of iodide by the thyroid gland through competitive inhibition of the iodide transporter. Their effectiveness is unpredictable and ClO4- can cause aplastic anemia, so these drugs are rarely used. The answer is A.

Infections caused by gram-negative bacilli have occurred when this cationic surfactant has been used as a skin antiseptic. (A) Acetic acid (B) Benzalkonium chloride (C) Lindane (D) Hexachlorophene (E) Thimerosal

Pseudomonas and other gram-negative bacteria have caused infections after the use of cationic surfactants such as benzalkonium and cetylpyridinium chlorides, partly because they form a film on the skin under which microorganisms can survive. In addition, some gram-negative bacilli are able to grow in solutions containing benzalkonium salts. Bacterial growth may also occur in solutions of povidone-iodine. The answer is B.

Concerning quinupristin-dalfopristin, which statement is accurate? (A) Active in treatment of infections caused by E faecalis (B) An effective drug in treatment of multidrug-resistant streptococcal infections (C) Bacteriostatic (D) Hepatotoxicity has led to FDA drug alerts (E) Increase the activity of hepatic drug-metabolizing enzymes

Quinupristin-dalfopristin is bactericidal against many drugresistant gram-positive cocci, including multidrug-resistant streptococci, MRSA, and vancomycin-resistant enterococci. The streptogramins have activity against E faecium (not E faecalis). The drugs are potent inhibitors of CYP3A4 and interfere with the metabolism of many other drugs. The streptogramins are not hepatotoxic. The answer is B.

Though rare, a serious toxicity associated with the thioamides is which of the following? (A) Agranulocytosis (B) Lupus erythematosus-like syndrome (C) Myopathy (D) Torsades de pointes arrhythmia (E) Thrombotic thrombocytic purpura (TTP)

Rarely, the thioamides cause severe adverse reactions that include agranulocytosis, vasculitis, hepatic damage, and hypoprothrombinemia. The answer is A

Which statement about the actions of antimicrobial agents is false? (A) Metronidazole has activity against C difficile. (B) Neonatal gonococcal ophthalmia can be prevented by silver nitrate (C) Polymyxins act as cationic detergents to disrupt bacterial cell membranes (D) Resistance to nitrofurans emerges rapidly, and there is cross-resistance with sulfonamides (E) Salicylic acid has useful antidermatophytic activity when applied locally

Resistance emerges very slowly when nitrofurantoin is used as a urinary antiseptic. There is no cross-resistance between the drug and other drugs used in the treatment of bacterial infections of the urinary tract. The answer is D.

Plasmodial resistance to chloroquine is due to (A) Change in receptor structure (B) Decreased accumulation of the drug in the food vacuole (C) Increased activity of DNA repair mechanisms (D) Increased synthesis of dihydrofolate reductase (E) Induction of drug-inactivating enzymes

Resistance to chloroquine in P falciparum can result from decreased accumulation of the drug in the food vacuole caused by the activity of a transporter system encoded by the pfcrt gene. The answer is B.

Which of the following is a unique property of SERMs? (A) Act as agonists in some tissues and antagonists in other tissues (B) Activate a unique plasma membrane-bound receptor (C) Have both estrogenic and progestational agonist activity (D) Inhibit the aromatase enzyme required for estrogen synthesis (E) Produce estrogenic effects without binding to estrogen receptors

SERMs such as tamoxifen and raloxifene exhibit tissuespecific estrogenic and antiestrogenic effects. The answer is A.

A young couple (25-year-old male, 23-year-old female) wants to start a family. They have not conceived after 1 yr of unprotected intercourse. Infertility evaluation revealed no abnormalities in the female partner and low sperm count in the male. Which of the following is a drug that is purified from the urine of postmenopausal women and is used to promote spermatogenesis in infertile men? (A) Desmopressin (B) Gonadorelin (C) Goserelin (D) Somatropin (E) Urofollitropin

Spermatogenesis in males requires the action of FSH and LH. Urofollitropin, which is purified from the urine of postmenopausal women, is used clinically to provide FSH activity. The answer is E.

Which statement about the clinical use of sulfonamides is false? (A) Active against C trachomatis and can be used topically for treatment of chlamydial infections of the eye (B) Are not effective as sole agents in the treatment of prostatitis (C) Effective in Rocky Mountain spotted fever (D) In some bacterial strains resistance occurs via increased PABA formation (E) Reduced intracellular uptake is a mechanism of sulfonamide resistance in some bacterial strains

Sulfonamides have minimal therapeutic actions in rickettsial infections. Chloramphenicol may be used for Rocky Mountain spotted fever in patients with established allergy or other contraindication to tetracyclines. All of the other statements about sulfonamide antimicrobial drugs are accurate. The answer is C.

A 50-year-old woman with a positive mammogram undergoes lumpectomy and a small carcinoma is removed. Biochemical analysis of the cancer reveals the presence of estrogen and progesterone receptors. After this procedure, she will probably receive which of the following drugs? (A) Danazol (B) Flutamide (C) Leuprolide (D) Mifepristone (E) Tamoxifen

Tamoxifen has proved useful in adjunctive therapy of breast cancer; the drug decreases the rate of recurrence of cancer. The answer is E.

The mechanism of antibacterial action of tetracycline involves (A) Antagonism of bacterial translocase activity (B) Binding to a component of the 50S ribosomal subunit (C) Inhibition of DNA-dependent RNA polymerase (D) Interference with binding of aminoacyl-tRNA to bacterial ribosomes (E) Selective inhibition of ribosomal peptidyl transferases

Tetracyclines inhibit bacterial protein synthesis by interfering with the binding of aminoacyl-tRNA molecules to bacterial ribosomes. Peptidyl transferase is inhibited by chloramphenicol. The answer is D.

The PPAR-γ receptor that is activated by thiazolidinediones increases tissue sensitivity to insulin by which of the following mechanisms? (A) Activating adenylyl cyclase and increasing the intracellular concentration of cAMP (B) Inactivating a cellular inhibitor of the GLUT2 glucose transporter (C) Inhibiting acid glucosidase, a key enzyme in glycogen breakdown pathways (D) Regulating transcription of genes involved in glucose utilization (E) Stimulating the activity of a tyrosine kinase that phosphorylates the insulin receptor

The PPAR-γ receptor belongs to a family of nuclear receptors. When activated, these receptors translocate to the nucleus, where they regulate the transcription of genes encoding proteins involved in the metabolism of carbohydrate and lipids. The answer is D.

Which statement is accurate regarding the antibacterial action of the aminoglycoside amikacin? (A) Antibacterial activity is often reduced by the presence of an inhibitor of cell wall synthesis (B) Antibacterial action is not concentration-dependent (C) Antibacterial action is time-dependent (D) Efficacy is directly proportional to the duration of time that the plasma level is greater than the minimal inhibitory concentration (E) The drug continues to exert antibacterial effects even after plasma levels decrease below detectable levels

The antibacterial action of aminoglycosides is concentration dependent rather than time dependent. The activity of amikacin continues to increase as its plasma level rises above the minimal inhibitory concentration (MIC). When the plasma level falls below the MIC, aminoglycosides continue to exert antibacterial effects for several hours, exerting a postantibiotic effect. Inhibitors of bacterial cell wall synthesis often exert synergistic effects with aminoglycosides, possibly by increasing the intracellular accumulation of the aminoglycoside. The answer is E.

In Graves' disease, the cause of the hyperthyroidism is the production of an antibody that does which of the following? (A) Activates the pituitary thyrotropin-releasing hormone (TRH) receptor and stimulates TSH release (B) Activates the thyroid gland TSH receptor and stimulates thyroid hormone synthesis and release (C) Activates thyroid hormone receptors in peripheral tissues (D) Binds to thyroid gland thyroglobulin and accelerates its proteolysis and the release of its supply of T4 and T3 (E) Binds to thyroid-binding globulin (TBG) and displaces bound T4 and T3

The antibodies produced in Graves' disease activate thyroid gland TSH receptors. Their effects mimic those of TSH. The answer is B.

Which statement about fluconazole is accurate? (A) Does not penetrate the blood-brain barrier (B) Drug of choice in treatment of aspergillosis (C) Induces hepatic drug-metabolizing enzymes (D) Has the least effect of all azoles on drug metabolism (E) Oral bioavailability is less than that of ketoconazole

The azoles with activity against Aspergillus are itraconazole and voriconazole. Fluconazole is the best absorbed member of the azole group by the oral route and the only one that readily penetrates into cerebrospinal fluid. Although fluconazole may inhibit the metabolism of some drugs, it has the least effect of all azoles on hepatic microsomal drug-metabolizing enzymes. The answer is D.

A patient with type 2 diabetes is taking metformin. The fasting glucose levels are in range, but the postprandial glucose is uncontrolled. All of the following drugs would be appropriate to add to metformin to target postprandial glucose except: A. Acarbose. B. Exenatide. C. Insulin aspart. D. Pramlintide.

The correct answer = D. Although all of these drugs target postprandial glucose, pramlintide should only be used in conjunction with mealtime insulin. Since this patient is not on insulin, pramlintide is not indicated.

A 76-year-old man is seen in a hospital emergency department complaining of pain in and behind the right ear. Physical examination shows edema of the external otic canal with purulent exudate and weakness of the muscles on the right side of the face. The patient informs the physician that he is a diabetic. Gram stain of the exudate from the ear shows many polymorphonucleocytes and gram-negative rods, and samples are sent to the microbiology laboratory for culture and drug susceptibility testing. A preliminary diagnosis is made of external otitis. At this point, which of the following is most appropriate? (A) Amikacin should be injected intramuscularly and the patient should be sent home (B) Analgesics should be prescribed, but antibiotics should be withheld pending microbiological results (C) Oral cefaclor should be prescribed together with analgesics, and the patient should be sent home (D) The patient should be hospitalized and treatment started with imipenem-cilastatin (E) The patient should be hospitalized and treatment started with gentamicin plus ticarcillin

The diabetic patient with external otitis is at special risk because of the danger of spread to the middle ear and possibly the meninges, so hospitalization is advisable, especially in the elderly. Likely pathogens include E coli and Pseudomonas aeruginosa, and coverage must be provided for these and possibly other gram-negative rods. The combination of an aminoglycoside plus a wider spectrum penicillin is most suitable in this case and is synergistic against many pseudomonas strains. Imipenem-cilastatin is also possible, but resistant strains of P aeruginosa have emerged during treatment. Cefaclor lacks antipseudomonal activity. The answer is E.

Which of the following drugs is most likely to cause hypoglycemia when used as monotherapy in the treatment of type 2 diabetes? (A) Acarbose (B) Canagliflozin (C) Glyburide (D) Metformin (E) Miglitol (F) Rosiglitazone

The insulin secretagogues, including the sulfonylurea glyburide, can cause hypoglycemia as a result of their ability to increase serum insulin levels. The biguanides, thiazolidinediones, α-glucosidase inhibitors, and canagliflozin are euglycemics that are unlikely to cause hypoglycemia when used alone. The answer is C.

An adult patient (weight 80 kg) has bacteremia suspected to be due to a gram-negative rod. Tobramycin is to be administered using a once-daily dosing regimen, and the loading dose must be calculated to achieve a peak plasma level of 20 mg/L. Assume that the patient has normal renal function. Pharmacokinetic parameters of tobramycin in this patient are as follows: Vd = 30 L; t1/2 = 3 h; CL = 80 mL/min. What loading dose should be given? (A) 100 mg (B) 200 mg (C) 400 mg (D) 600 mg (E) 800 mg

The loading dose of any drug is calculated by multiplying the desired plasma concentration (mg/L) by the volume of distribution (L). The answer is D.

Interactions between this drug and cell membrane components can result in the formation of pores lined by hydrophilic groups present in the drug molecule. (A) Caspofungin (B) Flucytosine (C) Griseofulvin (D) Nystatin (E) Terbinafine

The polyene antifungal drugs amphotericin B and nystatin are amphipathic molecules that can interact with ergosterol in fungal cell membranes to form artificial pores. In these structures, the lipophilic groups on the drug molecule are arranged on the outside of the pore, and the hydrophilic regions are located on the inside. The fungicidal action of the polyenes derives from this interaction, which results in leakage of intracellular constituents. The answer is D.

A 27-year-old woman underwent near total thyroidectomy. She was started on levothyroxine. What hormone is produced in the peripheral tissues when levothyroxine is administered? (A) Methimazole (B) T3 (C) T4 (D) TSH (E) FSH

The thioamides (methimazole and propylthiouracil) act in thyroid cells to prevent conversion of tyrosine residues in thyroglobulin to MIT or DIT. Levothyroxine (T4) is converted into T3 in the periphery. FSH is follicle-stimulating hormone. The answer is B.

The decision is made to begin treatment with methimazole. Methimazole reduces serum concentration of T3 primarily by which of the following mechanisms? (A) Accelerating the peripheral metabolism of T3 (B) Inhibiting the proteolysis of thyroid-binding globulin (C) Inhibiting the secretion of TSH (D) Inhibiting the uptake of iodide by cells in the thyroid (E) Preventing the addition of iodine to tyrosine residues on thyroglobulin

The thioamides (methimazole and propylthiouracil) act in thyroid cells to prevent conversion of tyrosine residues in thyroglobulin to MIT or DIT. The answer is E.

In an accidental needlestick, an unknown quantity of blood from an AIDS patient is injected into a resident physician. The most recent laboratory report on the AIDS patient shows a CD4 count of 20/μL and a viral RNA load of greater than 107 copies/mL. The most appropriate course of action regarding treatment of the resident is to (A) Determine whether HIV transmission has occurred by monitoring the patient's blood (B) Treat with a single high dose of zidovudine (C) Treat with full doses of zidovudine for 4 wk (D) Treat with single doses of zidovudine and indinavir (E) Treat with zidovudine plus lamivudine plus ritonavir for 4 wk

The viral RNA titer in the blood from the AIDS patient in this case is very high, and this needlestick must be considered as a high-risk situation. Although full doses of zidovudine for 4 wk has been shown to have prophylactic value, in highrisk situations combination regimens are favored. Optimal prophylaxis in this case might best be provided by the combination of zidovudine with lamivudine (basic regimen), plus the addition of protease inhibitors (expanded regimen). The answer is E.

A 3-year-old boy with failure to thrive and metabolic disturbances was found to have an inactivating mutation in the gene that encodes the growth hormone receptor. Which of the following drugs is most likely to improve his metabolic function and promote his growth? (A) Atosiban (B) Bromocriptine (C) Mecasermin (D) Octreotide (E) Somatropin

This child's condition is due to the inability of GH to stimulate the production of insulin-like growth factors, the ultimate mediators of GH effects. Mecasermin, a combination of recombinant IGF-1 and the binding protein that protects IGF-1 from immediate destruction, will help correct the IGF deficiency. Because of the inactive GH receptors, somatropin will not be effective. The answer is C.

Which of the following drugs is taken during the first part of a meal for the purpose of delaying the absorption of dietary carbohydrates? (A) Acarbose (B) Exenatide (C) Glipizide (D) Pioglitazone (E) Repaglinide

To be absorbed, carbohydrates must be converted into monosaccharides by the action of α-glucosidase enzymes in the gastrointestinal tract. Acarbose inhibits α-glucosidase and, when present during digestion, delays the uptake of carbohydrates. The answer is A.

This drug has characteristics almost identical to those of gentamicin but has much weaker activity in combination with penicillin against enterococci. (A) Amikacin (B) Erythromycin (C) Netilmicin (D) Spectinomycin (E) Tobramycin

Tobramycin is almost identical to gentamicin in both its pharmacodynamic and pharmacokinetic properties. However, for reasons that are unclear, it is much less active than either gentamicin or streptomycin when used in combination with a penicillin in the treatment of enterococcal endocarditis. The answer is E.

A 55-year-old patient with a prosthetic heart valve is to undergo a periodontal procedure involving scaling and root planing. Several years ago, the patient had a severe allergic reaction to procaine penicillin G. Regarding prophylaxis against bacterial endocarditis, which one of the following drugs taken orally is most appropriate? (A) Amoxicillin 10 min before the procedure (B) Clindamycin 1 h before the procedure (C) Erythromycin 1 h before the procedure and 4 h after the procedure (D) Vancomycin 15 min before the procedure (E) No prophylaxis is needed because this patient is in the negligible risk category

This patient is in the high-risk category for bacterial endocarditis and should receive prophylactic antibiotics before many dental procedures. The American Heart Association recommends that clindamycin be used in patients allergic to penicillins. Oral erythromycin is not recommended because it is no more effective than clindamycin and causes more gastrointestinal side effects. Intravenous vancomycin (not oral), sometimes with gentamicin, is recommended for prophylaxis in high-risk penicillin-allergic patients undergoing genitourinary and lower gastrointestinal surgical procedures. Complete cross-allergenicity must be assumed between individual penicillins. The answer is B.

A 27-year-old nursing mother is diagnosed as suffering from genital herpes. She has a history of this viral infection. Previously, she responded to a drug used topically. Apart from her current problem, she is in good health. Which drug to be used orally is most likely to be prescribed at this time? (A) Amantadine (B) Foscarnet (C) Ritonavir (D) Trifluridine (E) Valacyclovir

Three of the drugs listed (foscarnet, trifluridine, valacyclovir) are active against strains of herpes simplex virus. Foscarnet is not used in genital infections (HSV-2) because clinical efficacy has not been established, it has poor oral bioavailability and the drug causes many toxic effects. Trifluridine is used topically but only for herpes keratoconjunctivitis (HSV-1). Valacyclovir is converted to acyclovir by first-pass metabolism in the intestine and liver. The answer is E.

Oral formulations of this drug should not be used in a pregnant AIDS patient because they contain propylene glycol. One of the characteristic adverse effects of the drug is hyperpigmentation on the palms of the hands and soles of the feet, especially in African-American patients. (A) Amprenavir (B) Emtricitabine (C) Efavirenz (D) Fosamprenavir (E) Zalcitabine

Three of the drugs listed should be avoided, or used with extreme caution, in the pregnant patient. Oral forms of amprenavir and emtricitabine both contain propylene glycol, a potentially toxic compound. Efavirenz has caused fetal abnormalities in pregnant monkeys. However, one of the distinctive adverse effects of emtricitabine is hyperpigmentation. The answer is B.

Supplementary folinic acid may prevent anemia in folatedeficient persons who use this drug; it is a weak base achieving tissue levels similar to those in plasma (A) Ciprofloxacin (B) Levofloxacin (C) Linezolid (D) Sulfamethoxazole (E) Trimethoprim

Trimethoprim is the only weak base listed (fluoroquinolones and sulfonamides are acidic compounds), and its high lipid solubility at blood pH allows penetration of the drug into prostatic and vaginal fluid to reach levels similar to those in plasma. Leukopenia and thrombocytopenia may occur in folate deficiency when the drug is used alone or in combination with sulfamethoxazole. Fluoroquinolones and linezolid do not exacerbate symptoms of folic acid deficiency. The answer is E.

Trimethoprim-sulfamethoxazole is established to be effective against which of the following opportunistic infections in the AIDS patient? (A) Cryptococcal meningitis (B) Herpes simplex (C) Oral candidiasis (D) Toxoplasmosis (E) Tuberculosis

Trimethoprim-sulfamethoxazole is not effective in the treatment of infections caused by viruses, fungi, or mycobacteria. However, the drug combination is active against certain protozoans, including Toxoplasma, and can be used for both prevention and treatment of toxoplasmosis in the severely immunocompromised AIDS patient. The answer is D.

A 22-year-old man with gonorrhea is to be treated with cefixime and will need another drug to provide coverage for possible urethritis caused by C trachomatis. Which of the following drugs is least likely to be effective in nongonococcal urethritis? (A) Azithromycin (B) Ciprofloxacin (C) Erythromycin (D) Nitrofurantoin (E) Tetracycline

Urinary tract infections resulting from C trachomatis are likely to respond to all of the drugs listed except nitrofurantoin. However, nitrofurantoin is effective against many bacterial urinary tract pathogens with the exception of Pseudomonas aeruginosa and strains of Proteus. The answer is D.

Which statement about vancomycin is accurate? (A) Active against methicillin-resistant staphylococci (B) Bacteriostatic (C) Binds to PBPs (D) Hepatic metabolism (E) Oral bioavailabilit

Vancomycin is a bactericidal glycoprotein. It inhibits cell wall synthesis but does not bind to PBPs and is not susceptible to beta-lactamases. Vancomycin is not absorbed after oral administration and is used by this route in the treatment of colitis caused by C difficile and staphylococci. It undergoes renal elimination. Vancomycin is commonly considered the drug of first choice for parenteral use against methicillinresistant staphylococci. The answer is A.

Which statement about the mechanisms of action of antiviral drugs is accurate? (A) Acyclovir has no requirement for activation by phosphorylation (B) Ganciclovir inhibits viral DNA polymerase but does not cause chain termination (C) Increased activity of host cell ribonucleases that degrade viral mRNA is one of the actions of interferon-α (D) The initial step in activation of foscarnet in HSVinfected cells is its phosphorylation by thymidine kinase (E) The reverse transcriptase of HIV is 30-50 times more sensitive to inhibition by fosamprenavir than host cell DNA polymerases

ganciclovir inhibits viral DNA polymerase and causes chain termination. However, foscarnet inhibits viral DNA polymerase without requiring bioactivation. Fosamprenavir is the prodrug of amprenavir, an inhibitor of HIV protease; it has no significant effect on reverse transcriptase. The answer is C


Related study sets

Intercultural Communication Chapter 4-6 Key

View Set

Chapter 5 - Suitability, Replacement, and Compliance

View Set

Chapter 6 Health Assessment: Assessing mental status and substance abuse

View Set

Sociology Exam 2 (Practice Exams)

View Set

Chapter 2 Financial statements, taxes, and cash flow

View Set

Chapter 18: An Unsettled World, 1890-1914

View Set

Lección 2 Fotonovela Pronunciación . 4 - Dictado Audio InstructionsYou will hear a conversation. Listen carefully and, during the pauses, write what you hear. The entire conversation will then be repeated so you can chec

View Set

Paramedic National Test Prep: Medical

View Set